Remembered Questions for STEP 1 - USMLE Forum

December 18, 2016 | Author: eneropa | Category: N/A
Share Embed Donate


Short Description

Download Remembered Questions for STEP 1 - USMLE Forum...

Description

Remembered questions for STEP 1 - USMLE Forum

USMLE Forum Step 1



>>

#193828

nassab - 06/12/07 04:05

Step 2 CS Matching & Residency

This is the compilation or remembered questions. Many you might have it but thought it might help the new comers

Step 3 Miscellaneous

Archives

pharmocology: 1. SE of Cyclophosamide: h. cystitis 2. Tx of h. cystitis: MESNA 3. Tx of Pnumonia 4. MOA of Gancyclovir 5. MOA of Vincristine 6. MOA of Metotrexate 7. Prozosine...BPH, alpha1, HTN 8. Action potential curve...Ca++ channels closed 9. Cefataxime , indication 10. MOA of Glybride 11. Rifampin: tx for TB pt. 12. Asked x drug (warferin) MOA: acts like anti vit-K drug(choice) 13. Identify Partial antagonist on the graph 14. MOA of Colchicine 15. Tx for tonic clonic seizure? 16. Tx for a Kid with absent seizure beside ethosuximide? 17. Tx for a pt. with depression? Phenelzine 18. Furosemide : pick the correct area affected on the diagram of nephron. 19. Dopamin affects what NT? Ephineprine, NE, ... 20. What drug is given post MI? I picked Aspirine 21. Tx for sleep apnea? methylamphedamine 22. MOA of Cocaine 23. Ketoconazole? 24. MOA of AZT? 25. Tx for a pt with fever, productive cough, greenish sputum? 26. MOA of Rifampin? inhibits what assembly? 27. MOA and SE of Amphotrecine? 28. MOA of Gentamicine? Ristricts polysome formation 1. A pt has orchitis and has decreased libido how do you treat him? (Testosterone, hCG, I cann’t see the other choices) 2- a man and a woman have autosomal doiminant disease got married, If you know that they have natural stature, which disease of the following, their children have a big chance to get it: cystic fibrosis bachondroplasia c- MELAS. 3- what happened if the dashed part is affected (look at the diagram at Q3) many choices I choose loss of vibration an touch. 4- A girl got sore throat and was treated with penicillin without improvement, after sometime she died with heart problems, what is the cause? a. cocci b. CMV c. coxackie virus. 5- Viruses that carry virion associated polymerase what is the enzyme that is not found in the cytoplasm and this enzyme is used instead? a. DNA dependant DNA polymerase b. DNA dependant RNA polymerase c. RNA dependant RNA polymerase. 6- What do we need for PCR? a.deoxyribonuleotide b. DNA ligase. 7- How does mRNA exit the nuleouse? Transport dependant on ATP 8- What produce superoxide? a. superoxide dismutase b. NADPH oxidase c. catalase d. glutathione peroxidase

http://www.usmleforum.com/files/forum/2007/1/193828.php[11/7/2011 2:10:26 PM]

Remembered questions for STEP 1 - USMLE Forum 9- A boy has deficiency of skull mineralization what is affected? a. fibrillin b. chondroiin c. collagen d. heparin sulfate 10- A pt has kidney stones how do you treat him? a. mannitol b. furosemide c. spironolactone d. hydrochlorothiazide e. indapamide 11- Not clear 12- Not clear 13- MRI of brain with a mass in the ventricle this tumor most probably from? a. melanocyte b. epithelial cells c. meningioma 14- Q about apoptosis 15- A labeled draw of virus infecting a cell, asked about which level do the nucleoside RT inhibitors affect? 16- A pt had heart transplant and now is treated with cyclosporine, then he got respiratory problem with decreased lung functions what is the cause? a. CMV b. PCP 17- A pt has cardiac tamponade what do you see under microscope after two hours prove that it’s irreversible injury? A. acute swelling of mitochondria b. acute swelling of plasma membrane c. migration of nucleus 18- What is the most important factor for otitis externa? A. P.aeurginosa b. S. pneumoniae 19- A female with STD, she has microorganism covering skin cells and clue cells what is the diagnosis? A. candida b. trichomonas c. bacterial vaginosis 20- Two girls 16 and 14 years old came to you, the 14 years girl has STD what is your next step? A. treat her with permission of her older sister b. treat her without permission c. treat her with her parents permission 21- A pt with cancer in need to surgery, but his response is that â €œwhat is the benefit if I’m gonna die soon†your response is? A. ask him to go and get help b. show him cases with the same disease so he can agree for surgery. 11- Not clear 12- Not clear 13- MRI of brain with a mass in the ventricle this tumor most probably from? a. melanocyte b. epithelial cells c. meningioma 14- Q about apoptosis 15- A labeled draw of virus infecting a cell, asked about which level do the nucleoside RT inhibitors affect? 16- A pt had heart transplant and now is treated with cyclosporine, then he got respiratory problem with decreased lung functions what is the cause? a. CMV b. PCP 17- A pt has cardiac tamponade what do you see under microscope after two hours prove that it’s irreversible injury? A. acute swelling of mitochondria b. acute swelling of plasma membrane c. migration of nucleus 18- What is the most important factor for otitis externa? A. P.aeurginosa b. S. pneumoniae 19- A female with STD, she has microorganism covering skin cells and clue cells what is the diagnosis? A. candida b. trichomonas c. bacterial vaginosis 20- Two girls 16 and 14 years old came to you, the 14 years girl has STD what is your next step? A. treat her with permission of her older sister b. treat her without permission c. treat her with her parents permission 21- A pt with cancer in need to surgery, but his response is that â €œwhat is the benefit if I’m gonna die soon†your response is? A. ask him to go and get help b. show him cases with the same disease so he can agree for surgery. Viruses that carry virion associated polymerase what is the enzyme that is not found in the cytoplasm and this enzyme is used instead? a. DNA dependant DNA polymerase b. DNA dependant RNA polymerase c. RNA dependant RNA polymerase Re:"2007 Crush Bank" #653475 ericvorheese - 02/09/07 19:35

http://www.usmleforum.com/files/forum/2007/1/193828.php[11/7/2011 2:10:26 PM]

Remembered questions for STEP 1 - USMLE Forum great idea! here's my contribution...........an experience from a test that was taken earlier this week Biochem: some key enzyme deficiency ones (Lesch Nyhan, Maple Syrup Disease) but all of the questions were very obvious and did not require you to put much thought into it. Don't blow off porphyria and lead poisoning-- for some reason I got so many questions on that! Molecular Bio: This was a big one! lots of questions about DNA regulation, transcription, translation, bacterial plasmids, etc. Sometimes these questions look very scary -- they are always so long and use long names for molecules or restriction enzymes that you have never heard of. You need to get used to the question style and realize that what they are asking is very simple. The NBME forms have questions very similar to the molecular bio ones I saw on the exam. Pharm: Another one I thought would be difficult but not. Big drugs you should know a lot about (like antihypertensives, drugs for hyperlipidemias, cardic drugs, etc.) know side effects -- especially the chemo and immunosupressant drugs that kept on tripping me up so much. Micro/Immuno: mostly bacterial processes and what you would use to treat them... or what was their mechanism of disease (i.e. endotoxn, exotoxin, etc.) Know immunology and cytokines well, as well as the functions of all the cells. Different immune deficiencies were all asked on my exam (there is one page in First Aid that sums them up very well). Anatomy/Neuroanatomy: Always combined with a pathology question or an imagine. I had a few branchial plexus/lumbar plexus questions. Many questions would give you a clinical scenario, then ask you to identiy the artery/nerve/organ on a CT scan/MRI/angiogram/brain cross section. Physio: This was almost always combined with Pathology--they would ask the physiology behind some path process. I had so many questions where the question asked "what would be the levels of x, y, and z enzymes/hormones?", answer choices being "increased, decreased, etc" Pathology: lot of images -- MANY more than Qbank's representation. images from NBME test were repeated on my actual exam Goljan's book and lectures were great. He pointed out lots of things that wound up being on the exam and presents them in a way that really sticks. Also, his images are a great resource as i already gave 1 qs which i heard from 1 of my frnd the qs is a female birth a baby who died soon after birth and her placenta was edematous ,she has another child who goes to day care.wat is the diagnosis of the child death 1 ebv 2 cmv 3 parvovirus ans is parvovirus MLC MLO MLM AE -

- Most likely cause - Most likely outome - Most likely mechanism adverse effect

MLM Cytochalasin D - actin polymerization diaphragm innervation theophylline MOA Vincristine MOA Cyclophosphamide - AE Potters syndrome - emb connection Sarcoidosis - hypercalcemia MLM ADH MOA MLO paraxial mesoderm

http://www.usmleforum.com/files/forum/2007/1/193828.php[11/7/2011 2:10:26 PM]

Remembered questions for STEP 1 - USMLE Forum Paraneoplastic syn - SqCC Association MM - electrophoresis band pattern Apoptosis - blot pic WNT 11 gene - nephro development Retinoic Acid - MOA gp 120 abciximab AE Open Angle glaucoma - Rx Bells palsy ML cause from a list of 10 or so Isoasthenuria MLC Spousal abuse, next step HS - AD disease - inheritance pattern from pedigree Metanephrine, VMA are metabolites in Nblastoma and Pheochromo how would you differentiate. Blunt trauma - damage to speen CT scan - medial meniscus - locate X ray skull - pit fossa - Sheehan syndrome ACTH control over AC layers peroxidase - thyroid - HMP shunt connection Carbamoyl Phos contribution to NT structure Davenport diagram - R-Alkalosis As - Sq cell ca connection zona pellucida glycoproteins here is some stuff which i got just now 1 anaphylaxis reaction in a pt,gave epinephrine so wat mao of epi is? ans beta 2 effect 2 placebo & safety in which phase ?ans phase 1 3 turner synd pt has ammenorhea ,but she treted with estrogen /progesterone therapy,to have regular menseturation,she want to be pregnant what possibility she have?ans ova donation. 4 dantrolene moa 5 12 yr boy ,smooking cigratte and also steel mothers cigratte she is concered abt it?wat ans of a doc?a he send mom pap fpr smookingtherapy,b he tells boy abt lung ca c give boy busprinone 5 a pt have hypersensitivity bleeding easy brusing wat next compliction she have? ans was utrine rupture. 6 a pt have tetis and vagina a 5alpha redutase def ,B dihydrotestosterone def,c receptor respond ans c 7 predisposition ovarian ca 8 a female brt grand pa to doc ,he had alziemers,when doc chek he found stages of healing /ans was elder abuse 9 which heart diesease in turner? 10 64 yr old male ,bone pain, lytic lesion ?ans multiple fracture. 11 a 16 yr old grl bt inc, pt inc,which factor absent ans 8 12 moa of cholestyramine 13 in copd contraindicated drug is ? proparanol 14 asthma frst line of tretment? 15 2 qs abt dilated cardiomyopathy 16 5 qs abt b12 def 17 moa cisplatin 18 a women homeless live near by pesticide comp he sensory innervation goneand has stocking glove pattern skin infection?wat is the coz? a lead poisioning, b leprosy ans is b 19 a pt whose daughter took him to er ,tell abt she found near her father bed theophyline bottle, codine albuterol.thr is some more history ,wat treatment shld ? antidote naloxaone . Q) on how would you differentiate b/t Klebsiella from Salmonella A. Lactose fermentation Q) about Lesch Nyhan Syndrome- What Accumulates A. uric acid. Q)drug of choice for increased VLDL A. Niacin. Q) asks about what part of the Antibody binds to cells? A. FC portion Q) describes a child with eczema; low platelets and immune def- ask diagnosis A. Wiscott Aldrich Syndrome. . Dx of herpes, vesicular painful lesions 2. Conversion disorder: saying pt. paralyzed on one side 3. Schizotypal D/O: pt. lives with a lot of cats and is happy by herself 4. Sublimation defense mechanism: burned victum goes out for fund

http://www.usmleforum.com/files/forum/2007/1/193828.php[11/7/2011 2:10:26 PM]

Remembered questions for STEP 1 - USMLE Forum raising 5. Arteriogram of ACA, pt with lower exteremity paralysis 6. MRI of spine shown, pt can't feel pain and temp on lower exteremites 7. Esophageal veins initiates from what vein? 8. Gag reflex 9. Thymus anatomical location: between manubrium and ...hyoid bone? 10. 1st part of doudinum fed by what artery? 11. loss of dorsiflexion: what nerve? 12. what artery supplies testicles? where does it comes from? 13. what nerve supplies anal area? 14. Histologic pic shown: Dx? Diabetic GN 15. IgA def. pulmonary problems Dx? 16. Described skin infection: DX?HZV 17. Arrest of ovum? prophase I 18. 40 y/o undecended testes; his spermatogonia is in what phase? haploid? diploid? n? 2n? 19. Odd ratio: just simply know the formula. 20. Beta error , 1-B, 90%? (don't recall the whole thing. 21. Case control study: retrospective 22. Describe specifity 23. eyes can not do lateral gase, but looking down and up is ok. What nerve damaged? 24. pt can not look medially, what nerve damaged? 25. Renal cell carcinoma...vit D? 26. Thyroxine T4 activates what receptor? eg: thyrosine kinase? 27. Steroid receptors 28. PDA...cyanosis...continious murmur... at the Apex...>L hrt hyperthpy 29. Older pt. calcification of valves, vegitation? 30. Heme synthesis: option was: Succinyl CoA PX with signs & symptoms of goodpasture what would lm show on biopsy?cresent photo of plasma cella pt with lytic lesion vertebra wat disease? wat drug usually given pretreatment for leukemia moa of doxurubicin moa abacavir(ans reverse transcriptase inhibitor) h/o pt having gynecomastia ,lab:ast ,alt inc,....ans ketoconazol a Diag exactlly 4m frst aid ans chancre. staright qs abt BPH.ASKING qs abt endometriosis. pedigree of mito inheritance cleft lip failure of wat? case of alcholic hepatitis h/o UMN lesion identify the lesion MRI CASE SENSITIVITY AND SPECIFICITY pt having wt lose some psychtic symptom, dilated pupil asking abt drug abuse.. t 31. Rate limiting step in heme synthesis….ALA 32. steatourea…..b/c of HMG-CoA reductase 33. what is increased in lack of HMG-CoA reductase? Triglycerides? Chylomicrons? 34. pt. Macrocytic, neurological def…>B12 def 35. pt with gastritis, fundus region, anti-body against IF…what will be deficient? 36. B12 def …..terminal illium 37. Described Chron’s diz…asked lesions are where? Jujunium with bloody diarrhea 38. Rhumatiod arthritis 39. Pic of swallen finger; Septic arthritis? 40. pic of small petichia on skin…what is def? vit C def PT. in hot sun gets red on her face…b/c of? Options: Melanocytes,..? -ADA deficient: what cumulates? Options: Adenine, uric acid, urea? - MOA: Colchicine - pt. has low O2 sat…? 75% b/c of: options; 2,3BPG

http://www.usmleforum.com/files/forum/2007/1/193828.php[11/7/2011 2:10:26 PM]

Remembered questions for STEP 1 - USMLE Forum decreased,… - #’s given, DM1, ketoacidosis, Metabolic Alkolysis and acidosis. - post-op 7 days, MRI shown….option: PE - UTI older pt. , no Ecoli in the options…..but Gdiplocco?// - Picnic meat, watery dia…what bug? Options: camplobacter juju, strep saphraticus, c. basillus ? - Pt. With bloody diarrhea what bug ? - umbilical vein or artery thrombus to where causes cerebral infarct why? Options: open foramen secondum?... - V/Q mismatch: options: perfusion decreased at apex, or higher O2 ….KNOW ENDOCRINE PHYSIOLOGY AND PATHOPHYSIOLOGY …… Know your vitamins very well. .....Immuno-know all the cytokines ....neuro - know the cerebral cortex very well,all the areasbrocas,wernickes etc many questions on spinal cord segments.most of the neuro questions were based on images ....a nurse use iodine before giving insulin why--to sterlize that area from virus,bacteria and fungi ......a girl has 28 days cycle..physcian do her hormone test...on the day 26 what hormone will be increase ---option are FSH,LH,TSH,HCG ......MAKE SURE HOW TO READ CT SCANS AND MRI'S. THE ANATOMY QUESTIONS HAD ALOT OF CT SCANS SHOWING VARIOUS MUSCLES. Like for example, some person had a superior gluteal nerve lesion, and it had a CT scan of the pelvis, and you had to identify the MUSCLE. Also, .......2 QUESTIONS ON EPIDURAL HEMATOMA: ASKED WHICH ARTERY IS DAMAGED ......FETAL ALCOHOL SYNDROME: KNOW THE SIGNS!! (DON'T GET CONFUSED WITH DOWNS SYNDROME) .....HYPER/HYPO THYROIDISM 1)Pituitary tumor on visual fields. PITUTARY TUMOUR IS CRANIOPHRANGIOMA CAUSES BITEMPORAL HEMIANOPSIA. 2)Tumor anterior to temporal fossa affects wot? 3)Tends to fall to one side tongue deviates to other, site of lesion? .THIS is problem in 12 cranial nerve . which is locted in lower medulla. 4)Pediatric infratentorial tumor. .most of the paediatric tumours are infra tentorial.they are celleblar, astrocytoma.2.medulloblastoma.3.gliomas 5)Midline neck mass at level of hyoid. .BRANCHIAL CYST 6)Esophagal hiatus of diaphragm. .IT OPENS AT THE LEVEL OF T10, IVC hiatus at T8 descending aorta opening at T12 7)Splenic venous drainage. .VENOUS SINUSOIDS IN CORDS OF BILROTH 8)Ambiguous genitalia. .AMBIGOUS GENITIAL is in female with excess of testosterone or in male with lack of testorone. 9)Melanosis coli. .MELONOSIS COLI I is due over use of laxatives.colon appers black in colour. Melanosis coli is due to use of laxatives not antibiotics. Usually anthraquinone ( senna and cascara) 10)Inhalational antigen stimulation leading to antibody recovery from vaginal mucosa of experimental animal. How? It is called "Dissemination of Immunologic Memory" , and it is the same mechanism used in Adenovirus Vaccine. The adenovirus vaccine used by the military against adenovirus types 4 and 7 is an enteric-coated , live , non-attenuated virus preparation. This vaccine produces an asymptomatic intestinal infection and thereby induces mucosal IgA memory cells. These cells then populate the mucosal immune system throughout the body. ( Because Memory cells tend to home in a tissue-specific fashion , presumably returning to the type of tissue in which they first

http://www.usmleforum.com/files/forum/2007/1/193828.php[11/7/2011 2:10:26 PM]

Remembered questions for STEP 1 - USMLE Forum encountered antigen.) Vaccine recipients are thus protected against adenovirus acquired by aerosol , which could otherwise produce pneumonia. 11)x-ray foot with calcification parallel to first metatarsal with decreased dorsalis pedis. .myositis ossificens 12)Ligament that holds tibia from goin back: ant and posterior cruciate ligments--that holds tibia from going back is Posterior crusiate ligament ----ligament that prevents tibia from coming forward is Anterior cruciate ligament 13)Obturator nerve section: effects? obtutaror nerve injury leads to loss of adduction of thigh 14)Pelvic ascites, site of drainage on standing position. the site of drainage is vescicouteric pouch in females and rectovescical poch in males. 15)Aortic aneurysm rupture: effect on urinalysis in the background of anuria. aortic aneurysm leads to renal infarction.so there will rbc in urine. 16)Hand decreased sensation lateral aspect. median nerve injury 17)Hand sensation ok, but extention of metacarpophalangeal joints difficult. exention and flexion are done by lumbricals and interossis muscles. the extension at MP joint by-->extensors of the fingers flexion at MP joint by--->lumbricals and introssei. so if extension is difficult means extensors are weak...which are supplied by radial nerve... 18)Clubbing description: choices bronchiectasis, asthma clubbing is seen in bronchiectasis Q1) Cystic fibrosis heterozygous frequency? Q2) Chemotherapy heart failure – left heart/right heart/myocardial fibrosis? Maybe Doxorubicin (Adriamycin) FA p.330 or Daunorubicin . But have no idea what is the mechanism Q3) Stimulants and inhibitors of pancreatic secretions: exocrine and endocrine. Exocrine- Stimulants : CCK , Secretin , parasympathetics, Inhibitors: None EndocrineInsulin Stimulants : Glucose , Amino Acid , GIP , Glucagon, Insulin Inhibitors : Somatostatin , Sympathetics (Alpha) Glucagon Stimulants : Amino acid, Glucagon Inhibitors : Insulin , Somatostatin Q4) Cellular barrier between gall bladder cavity and wall’s epithelial cells. is maybe asking for Tight junction. Tight junction must be there to protect the internal celluar structures from nasty bile Q5) Embryology neural tube formation, cellular dna fragmentation indicates? Apoptosis ? Q6) War veteran describing killing business like – defence mech? Q7) Trigeminal nerve nucleus identification in medulla ct cross section.( you can just say verbally where its located) Q8) Efferent renal constriction: effects on GFR & Filtration fraction. GFR & FF both increases Q9)Premature birth, surfactant and steroid therapy fails due to surface tension increase/decrease etc. Q10) Barret esophageal cancer type? Adenocarcinoma , since barrett's is Metaplasia full of intesinal cells Q11)GERD cuz of secretions from which type of gastric cells? BRS Physiology(p.223) says that the relaxaton of lower esophageal sphincter is vagally mediated , and the neurotransmitter is VIP. Or, maybe GERD could be due to gastric acid therefore , parietal cell could be an answer. Q12) Cholecystectomy patients absorb fat from? Cholecystectomy pt. will still absorbing fat from terminal ileum I think, well ofcourse from small intestine cholecystectomy has nothing to do with the site of absorbtion . the person still produces bile( in liver) just no storage or conecentration of bile.... thats why post op cholecystectomy patients are put on low fat diet

http://www.usmleforum.com/files/forum/2007/1/193828.php[11/7/2011 2:10:26 PM]

Remembered questions for STEP 1 - USMLE Forum but site of absorbtion remains small intestine (ileum) as usual Q13)Sputum silver staining for bugs. silver staining reminds me of two bugs,but I am not sure : Pneumocysyis carinii , Legionella pneumoniae. But silver staining of PCP is done on pt.'s Bronchoalveolar lavage , I guess. Q14) First thing to say to dad patient brought by 2 daughters who express wish to remain with patient. Best statement to make to build rapport with patient playing crossword on encounter. May be Doctor have to ask the pt. whether he wants their daughter in the room while interviewing Q15) Post break-up with boyfriend, mother of patient finds synthroid tablets in nursing student daughter’s belongings. Doc’s response to patient? synthyroid is Thyroid hormone I guess, but not sure if it is T3 or T4. the pt. may be planning on weight loss because boyfriend hated her obesity . q16)Fragile X family, parents concern regarding testing of ?normal 14year daughter’s genome for similar problems. Q 17) Girl child dumps classes since uncle’s death, and mother finds it helpful for daughter to be around home. Intervention required? Family therapy/psychotherapy…etc. Q18) Porphyrias: like the back of ur hands. q19) Carbon tetrachloride: how it affects liver. CCl4 - The liver converts CCl4 to a CCl3 free radical , which damages the liver (fulminant hepatitis , fatty change) from Goljan's STARS pathology review p.93 Acyclovir mechanism of action. inhibhits DNA viral poymerase and phosphorylated by viral thymidine kinase Q20) Hernia above n lateral to pubis. Hernia above the inguinal lig. is Inguinal hernia , and below the lig. is femoral hernia. So it could be both Indirect and direct inguinal hernia since it doesn't say anything about hernia's relation to inf. epigastric vessels. Q21)Hydatidiform mole: learn how to interpret genotypes of moles Which 2 Joints are Never affected in RA ? Distal interphalangeal and the Back joints r never involved in RA Q1) Best means of bringing down temperature of seizing patient with meningitis: pharmacologic and otherwise. think non pharamacologic tt shd be hydrotherapy to bring down temp...n pharamacologic may be paracetamol by rectal route. Q2) Acetaminophen overdose: how it damages. In overdose, the stores of GSH are depleted and and the metabolite N-acetylbenzoquinoneimine reacts with hepatocytes leading to liver necrosis: Acetaminophen forms free radicals in the liver and results in fulminant hepatitis/ renal papillary necrosis. damages by free radicals which r removed by glutathione..N ACETYLCYSTEIN its antidote restores the glutathione levels Q3) Antimicrosomal antibodies. involved in hashimoto's autoimmune thyroiditis and autoimmune hepatitis Q4)Dialysis patient on transplant list gets angry often and misses dialysis appointments - intervention: need to spend more time with patient/discuss regarding reconsideration of transplant list…etc. Dialysis patient on transplant list gets angry often and misses dialysis appointments - intervention: need to spend more time with patient/discuss regarding reconsideration of transplant list…etc. ??? need to spend more time with patient to know what exactly is his prob Q5) Adenovirus – structure? nonenveloped,ds linear DNA, icosahedral nucleocapsid, only virus with a fibre protruding from each of 12 vertices of teh capsid. Q6) Brown fat of neonates. Function? site of heat production in a neonate---brown adipose tissue contain natural uncoupling protein, which allows energy loss in the form of heat to maintain basal temp Q7) Longterm steroid use: effect on adrenals. adrenal suppression... therefore tapered slowly, neg feed back on ant pit so, decreased ACTH, so less stim of adrenals…hypothalamo pituitary axis and eventually leads to atrophy of the adrenal gland. Q8) Trypanosome cruzi: life cycle. try cruzi lifecycle involves reduvid bud as a vector and both humans and animals as reservoirs. life cycle. … reservoirs are cats, dogs… reduviid bug passes the trypomastigote as it bites and scratching implants in bite site……….reservoir is rodents, armadillos. vector is reduviid bug. it transmits the trypomastigote form which turns into the amastigote form inside the body. At the bite site the lseion is called chagoma. Systemic symptoms are

http://www.usmleforum.com/files/forum/2007/1/193828.php[11/7/2011 2:10:26 PM]

Remembered questions for STEP 1 - USMLE Forum fever, lymphadenopathy . Tachycardia, meningoencephalitis. Then comes the intermediate phase where the patient is asymptomatic, with low levels of the parasite and antibodies against it. decades later-the chronic form emerges with dilated cardiomyopathy, megacolon, megaesophagus.Treatment is Nifurtimox. Q9)Patient requiring dialysis says: don’t want machines to keep me alive. Doc’s next step? Q10)Antifungals: big time. Drug interactions. Mechanism of actions.(lets write few antifungals with mech of action and main side effects...we will never forget it if we do so) amphotericin B binds ergosterol creates pores. flucytosine-inhibit thymidilate synthase. azole-inhibit ergosterol synthesis griesofulvin-interfere with microtubule function terbinafine- inhibit squalene epoxide DRUG INERACTIONS. AZOLE GRP INCREASE TOXICITY OF FOLL BY INHIBITING ITS METABOLISM bdz,cisapride,cyclosporine,fluoxetine,lovastatin,omeprazole Q11) Alcoholic with nosebleed. Wot to expect in terms of AST, PT, and forgot…( I dont know exactly whats being asked...lets see who figures it out) AST is increased, platelet count is decreased, prolonged PT Q12)Picture of normal hand and a slender hand shown and asked for diagnosis. Q13)Wernicke mammillary body identification on CT( here just name the structures affected in wernickes encephalopathy and ofcourse do know how to identify them...very very HY) mammillary bodies and dorsomedial nucleus of thalamus….Mamillary bodies on the ventral surface of the brain stem Q14) How HBV causes liver cancer. HBV DNA integrates in host genomic DNA,MODIFICATIONOF GENE EXPRESSION. HBV INCORPORATES itself into the hepatocyte dna and triggers malignant growth. theres is a 200 X Risk of developing HCC in carriers versus noncarriers Q15)Patient taking OCP. Smoker. Wot u wanna check first? smoking induces met enzymes increased metabolism of OCP'Sdecreased efficacy, Smoking itself is thrombogenic, and estrogen also is… ask for history of thrombosis, may be check PT, PTT Q16)Allopurinol – effect on renal stones? It inhibits xanthine oxidase, decreasing uric acid allopurinol used in pts of renal stones. as calcium oxalate stone formers are hyperuricemic Q17)Radiation alopecia – dermal histopathology? sparse mixed dermal inflammatory infiltrate consisting mainly of lymphocytes. . Mucinous deposits, confirmed by alcian blue/PAS stain, were observed within the epidermis of the hair follicle Q18)Streptococcus pyogenes – patient penicillin allergic. Next step? Can give Clavulanic acid, Sulbactam, Tazobactam. Cephalosporins should be avoided in patients allergic to penicillin because of cross sensitivity.. Cephalosporins are susceptible to beta lactamase, but less than penicillin. ? penicillin allergic pts erythromycin or azithromycin are the alternatives, all GRP A Streptoccoci r susceptible to pencillin G, but neither Rhematic fever nor allergic pats benefit from pencillin treatment after onset. In pencillin allergies pats should get erythromycin or azithromycin. .HOWEVER ENTEROCOCCAL ENDOCARDITIS CAN BE ERADICATED ONLY BY PENICILLIN OR VANCOMYCIN COMBINED WITH AN AMINOGLYCOSIDE" Q19)t-Rna triplet codon function? The anti codon on the tRNA pairs with the codon for aa in mRNA Q20) Nuclear membrane – evolutionary importance between eukaryotes and prokaryotes? all I know is nucleus membrane is absent in prokaryotes and present in euk may be evolutionary imp is to protect the genetic information...??? there are 4 trigeminal nuclei---> 1)primary sensory nuclei for sensations of face 2)spinal trigeminal nucleus for pain and tempreature sensation of face 3)Mesencepahalic nucleus for proprioception 4)primary motor nucleus for muscles of mastication

ZEBRA genes:Z EBV replication activator (ZEBRA) as lytic cycle

http://www.usmleforum.com/files/forum/2007/1/193828.php[11/7/2011 2:10:26 PM]

Remembered questions for STEP 1 - USMLE Forum markers.ZEBRA is one of the immediate early genes of EBV.it is considered to be expressed in the initial stage of the lytic cycle, before the viral particles have been dispersed. Zebra sign:. Hemorrhage is often characterized by a typical, streaky bleeding pattern due to blood spreading in the cerebellar sulci. this zebra-pattern hemorrhage seems to be typical in a postoperative loss of CSF. Zebra bodies - lipid inclusions with concentric lamellar structure with alternating light and dark-staining bands in neuronal cells in neimann picks Zebra tumor = acoustic neuroma Q1)Lead poisoning( symptoms) --Lead poisoning….. lead inhibits ferrochelatase and ALA dehydrase….., coproporhyrin and ALA accumulate in urine, symp—abdominal colic, sideroblastic anemia, wrist and foot drop, lead lines in gingivae and epiphysis of long bones, encephalopathy, basophilic stippling Q2) Glutamine in urea cycle Q3) Large intestine gross section: tumor shown. Type? (felt like the napkin constriction)...I never heard of this kind of tumor...anyone knows? Q4) Radiation therapy pelvis. Cause for urinary retention. ???cystits fibrosis Q5) Osteogenesis imperfecta: defect? d? AD, mutation in collagen genes Q6) Medial side of arm numbness after mastectomy: nerve involved? intercostobrachial n. also known as the lateral cutaneous br. of the ventral primary ramus of T2 Q7) Interleukins: big time. Learn function of each!!we can name some...and function. IL1- stim produced mainly by macrophages IL2 syn , pyrogenic, activates TH, IL2- downreg TH2, Tcell growth factor, stim Bcell IL3- stim BM stem cells IL4-class switch to IgE IL5-Class swish to eosinophil, promote B cell prolif IL6- produc of acute phase reactants IL8-chemotactic and adhesion of neutrophils IL10-Inhibit TH1, inhibit cytokine from macrophage IL12-induce TH0 to TH1 Q8) Plummer Vinson: esophageal defect? Esophageal webs… iron deficiency anemia, o Stomatitis o Glossitis o Dysphagia o Spoon-shaped nails o Esophageal webs Q9)Scurvy: enzyme/reaction involved? Prolyl and lysyl hydroxylase, the enzymes that catalyzes the hydroxylation, require vitamin C. Q10)Reiter syndrome vs. Sjogren... Reiter syndrome--- urethritis, conjunctivitis and ant uveitis, arthritis Sjogren...Xerophthalmia, xerostomia, arthritis, parotid enlargement Q11)Managing hypertensive crisis in MAO inhibitor patient on cheese: it’s pathogenesis? Treatment ot a Hypertensive Crisis from MAOI Reaction. This is a medical emergency with cerebral vascular accidents being one of the greatest concerns. Medical consultation is warranted. Patients should report to an immediately to an emergency department. Initial treatment options include: Phentolamine 5 mg IV or Thorazine 50 to 100 mg po Care should be taken to ensure that the patient does not become dangerously hypotensive or that the hypertensive state returns once the intervention medication has worn off. Tyramine is actively transported into neurons and displaces NE, leading to intraneuronal release of NE. It can be degraded by MAO, but if you are on MAOI, its interaction will cause tyramine build up. Tons of NE will be replaced and hence the HTN crisis. Q12) Neurotransmitter involved in Huntington. Decreased GABA and ACh Q13)Aortic coarctation: site of constriction and symptoms. infantile type is preductal, Cardiomegaly, Pulmonary venous congestion , Right Ventricular Hypertrophy Adult is postductal asso with notching of ribs, Displaced esophageal shadow rightward, Left Ventricular Hypertrophy, HT in UL, weak pulses in LL

http://www.usmleforum.com/files/forum/2007/1/193828.php[11/7/2011 2:10:26 PM]

Remembered questions for STEP 1 - USMLE Forum Q14) Chronic antiplatelet drug in aspirin sensitive? Clopidogrel & Ticlopidine : They are back-up or alternative drugs to aspirin. They block ADP receptors on platelets , hence decrease activation Q15)Stroke patient getting MI treated with streptokinase dies due to cva. Cause? concluded that death of the patient occurred due to rebleeding on administration of streptokinase 1. Intracranial Hemorrhage : As PRS has mentioned , the death could be due to intracranial hemorrhage. Since CVA refers to both ischemic (80%) and hemorrhagic(20%). This is more likely than other explanation to me. 2. Therapeutic Failure : It could be due to strptokinase's property itself. Since most individuals have had a streptococcal infection sometime in their lives , circulating antibodies against streptokinase are likely to be present in most patients. These antibodies can combine with streptokinase and neutralize its fibrinolytic properties. (Therapeutic failure) (lippincott Pharmacology 2nd edition p.203) 3. Property of all Thrombolytic agents As the thrombolytic agents dissolute clot , Increased local thrombin may occur as the clot dissolves , leading to enhanced platelet aggregability and thrombosis. Strategies to prevent this include administration of antiplatelet drugs , such as aspirin , or antithrombotics , such as heparin. Drugs that have been implicated in the development of serum sicknesslike reactions include the following: allopurinol (Zyloprim), arsenicals and mercurial derivatives, barbiturates, captopril (Capoten), cephalosporins, furazolidone (Furoxone), gold salts, griseofulvin (Fulvicin, Grifulvin), halothane, hydralazine (Apresoline), iodides, methyldopa, para-aminosalicylic acid, penicillamine, penicillins, phenytoin (Dilantin), piperazine, procainamide (Procan SR, Procanbid, Pronestyl-SR), quinidine (Quinaglute, Quinalan, Quinidex, Quinora), streptokinase (Streptase, Kabikinase), sulfonamides, and thiouracils. Q16)Leukotriene inhibitors? Zafirlukast…LT receptor antagonist Q17) Chromolyn sodium. Mechanism of action. prevent degranulation of mast cells Q18)Grave’s pathophysiology. autoimmune TSH receptor antibodies, type II hypersensitivity Q19) Couple in for infertility counseling. High BMI Wife says, “it kills me not to be able to have a baby.†Doc’s best response? obese wife possibly has Polycystic Ovarian Syndrome (PCOS). 74% of PCOS pt. has infertility and PCOS accounts for 30% of overall infertility. Therefore , the doctor may have to evaluate for PCOS by Lab. blood test and Ultrasonography. The lab blood test will show Increased androgen , LH , fasting insulin , prolactin and estrogens. The ultrasonography is the most sensitive diagnostic study. If the Dx. of PCOS have been confirmed , Tx should be Weight reduction , quit smoking , and antiandrogen(because of hirsutism) and antiestrogen(clomiphen) Q20)Baby awakens at night frightened, goes back to sleep. Does not respond to questions b parents on that awakening. Best next step? The boy is possibly having Night Terrors. It occurs in Delta sleep , so you can't wake them and they can't remember what happened. And night terror is shown to be precursor to temporal lobe epilepsy. The doctor's response should be either examine for any epilepsy in the pt.(prevent or tx. the temporal lobe epilepsy) or ... Identify & dealing with waking-time anxiety to relieve night terrors celiac disease is the proximal bowel and tropical sprue involves the entire bowel.. Q1) Prematue ejaculation. Best next step? Ans- selective seratonon reuptake inhibhitors like fluoxetine or even clorpramine (which is the most seratonin specific hetrocyclics) Q2)Delirium: big time. ( just define) Ans-Impaired conciousness(incontrast to dementia which is loss of

http://www.usmleforum.com/files/forum/2007/1/193828.php[11/7/2011 2:10:26 PM]

Remembered questions for STEP 1 - USMLE Forum memory and intellectual abilities) causes-Huntingtons or parkinson's ds, CNS infection,trauma, systemic ds( like hepatic, cardiovascular), high fever substance abuse or withdrawal. more common occurrence in children and elderly. associated physical findings-autonomic dysfunction acute medical illness amd ABNORMAL EEG. associated psycho findings-illusions hallucinations impaired conciousness sundowning(symptoms worse at night) treatment- removal of underlying cause will allow symptoms to resolve Q3)Malingering vs factitious. Ans-factitious.-when individual tries to simulate an illness for attention from medical personnel and can even undergo unnecessary medical and surgical procedures. Malingering-when the same is done for financial or other obvious gain. Q4)Leuprolide mechanism of action. ans- its a GnRH agonist and causes continued secretion of Gn from ant pituitary which causes the downregulation of Gn receptors--->inhibhition of FSH and LH-->supressed gonadal function. used in treating precosious pubery, , prostatic Ca,endometriosis, premenopausal breast Ca,utrine leomyoma, PCOD,....can also be used as contraceptive in both males and females Q5) Thromboangitis obliterans.( discuss in afew lines) Ans-also called buerger"s ds.affects most commonly smokers..it affects the whole neurovascular bundle of tibial poplteal or radial arteries. the thrombus is composed of neutrophil containing micro abscesses.Pts frequently have raynaud's phenomenon Q6)Lack of T tubules in muscles lead to…? I think lack of T tubules in muscle will affect the membrane depolarization and hence contraction. Q7)Tryptophan/tyrosine metabolism both of them are both ketogenic and glucogenic amino acids Q8) phosphofructokinase – inhibitors? ATP and citrate Q9)Lysosomal storage disorders...just name and imp charecteristic ans-fabry's and hunters are XR rest all are AR. Tay sachs-def of hexosaminidase and GM2 gangliosides accumulate. fabry's - def of alfa galactosidase and ceramide trihexoside accumulates. Mertachromatic leuko dystrophy- def of arylsulfatase and sulfatides accumumalate krabbes- beta galactosidase deficient and galactocerebroside accumulates Nieman pick-sphingomylinase def and sphingomylin accumulates. gaucher-def of beta glucosidase and glucocerebroside accumulates Q10).S-100 positive slide, prognostic factor? S100 is a tumor marker for malig. melanoma and imp factor which determines prognosis is depth of the lesion Q11)Gaucher bone marrow. Ans-gauchers cells present which are macrophages with wrinkeled paper cytoplasm Q12)Young boy with breast biopsy for post-traumatic swelling showing multilayered cells lining ducts: next step? maybe this is case of gynecomastia...its mostly noticed by medical personnel after trauma...In most cases, no treatment is needed,gynecomastia goes away in less than 3 years.Occasionally, medicines may be used to treat gynecomastia, especially if tenderness is a problem. treatment includes testosterone gel or surgery Q13)S shaped bug, curved bug. curved is vibrio...does S shape also include in this genus Im not sure Q14) Boy and pet dog both having diarrhea. No ova/parasites. ? q15) Scleroderma: cause for pulmonary hypertension? Ans-PH is high blood pressure in the arteries which take blood between the heart and lungs. When PH occurs along with other lung, heart, or systemic connective tissue disease (such as scleroderma), it is called Secondary Pulmonary Hypertension.when its due to scleroderma the cause maybe fibrosis Q16) Pulmonary hypertension patient listed for lung transplantation. Pharmacologic management during waiting period? maybe diuretics and oxygen therapy. looked up a website which stated epoprostenol(FLOLAN) for those listed for lung transplant Q17) Vascular bed, blood flow increased, howz capillary flow controlled? this occurs by arteriolar constriction to maintain a constant flow in capillaries. Q18) Carcinoid lung....(some imp features) Bronchial carcinoid tumors arise from Kulchitsky cells (argentaffin cells) within the bronchial

http://www.usmleforum.com/files/forum/2007/1/193828.php[11/7/2011 2:10:26 PM]

Remembered questions for STEP 1 - USMLE Forum mucosa.These cells are neurosecretory cells,which belong to APUD sysyem.They have the capacity to synthesize serotonin,ACTH,norepinephrine, bombesin, calcitonin, antidiuretic hormone (ADH), and bradykinin. The clinical manifestations of bronchial carcinoids may arise because of their endobronchial or central location, their potential for metastatic spread, or their ability to produce vasoactive amines. Hemoptysis is common, occurring in at least 50% of patients. This finding reflects the vascular nature of these lesions. patients may present with complications due to the neurosecretory activity of the tumors. Bronchial carcinoids may secrete ACTH in quantities sufficient to cause Cushing syndrome in 2% of patients Q19)Common cause for cataract and aged skin wrinkles? Q20) Retinoblastoma, risk for another cancer? ans-Some, especially those in which the tumor involves both the eyes, are at a risk for developing other tumors like osteogenic sarcoma....maybe due to the radiation therapy.. but I also had read some where that retinoblastoma patients are at increased risk of developing some brain tumors( cant reme which specific brain tumor)...plz correct me if Im wron 1.Testicular CA drains to ? para aortic LN 2.Meckel’s is a remnant of? vitellointestinal duct 3.A child with tumor near 3rd vent will present with what symp? obstructive hydrocephalus 4.A pic of carotid arteriogram with arrow pointing to one branch, asked for symp in pt due to its block MI/angina symptoms 5.MRI of abd given, asked to identify structure lying in relation to pancreas head? 2nd part of duodenum,bile duct 6. While operating for hyper PTH, surgeon finds 2 supr and 1 infr lobe, to locate the ectopic lobe she has to trace which artery? maybe inferior parathyroid artery 7. Histamine is predominantly released from ?? mast cells 8. Pt has loss of afferent limb of papillary reflex, asked to identify the structure in a cut section aferent limb of pupillary reflex is via optic nerve...so that must be the structure asked to identify 9. A neonate is for corrective cardiac surgery, to reach PDA catheter passed from femoral vein must pass throu--?Lpulm art, LA,RV,RA femoral vein-rt atria-pulm trunk-left pulm artery-PDA 10. Pt has spastic paralysis, asked to locate the area responsible on a gross pic of brain spastic paralysis is due to UMN lesion..so maybe the pyramidal tract...or higher areas in motor or premotor cortex 11. If sup mesenteric art is occluded at its origin and no sig collaterals, then which organ is most affected ?duo, ileum, caecum, colon 12. Pt with seizures and anosmia has which cerebral lobe affected anosmia indicates involvement of pyriform cortex( responsible for smell)....this is located in cerebellum...but quesasks which cerebral lobe involved 13. Hypertensive pt with head injury in lucid, later deteriorates brain CT given, asked if it is epidural, intra cereb, .. lucid interval indicates that its epidural 14. Asked for nerve supply to ant ½ of ext ear canal ant 1/2 of ext ear canal-supplied by auriculotemporal n post 1/2 by auricular br of vagus facial surface of upper part of auricle-auriculotemporal cranial surface of upper part of auricle-lesser occipitla n both surfaces of lower part of auricle-great auricular nerve 15. A 21yr male with acute LLQ abd pain & fever with vomiting, what other signs will be present-tenderness at McBuneys, / pain on passive flexion of hip? 16. Pt with stone in parotid duct, asked for thro which muscle does duct pass to open in oral cavity- zygomaticus major, temporalis, masseter, buccinator, orb oris? 17. A druken pt sleeps on arm chair develops wrist drop- nerve ? radial

http://www.usmleforum.com/files/forum/2007/1/193828.php[11/7/2011 2:10:26 PM]

Remembered questions for STEP 1 - USMLE Forum nerve injury 18. 10 yr child with elbow dislocation would also have damage to-? Deep brachial art, median N, interosseous memb, brachial v median nerve injury 19. Microscopic pic of FT with ovum in the process of fertilization, plenty of sperms also seen around, asked to identify the struc derived from glycoproteins?? 20. A girl with well dev breasts, pubic hair, has normal female ext with testes, neither male nor female int organs, what is the cause androgen insensitvity Adie's Pupil Dilated pupil which may react better to near than to light. It is probably due to disease affecting the ciliary ganglion. Is hypersensitive to any weak Pilocarpine (eg. 0.1%) with constriction of the pupil, in contrast to the pharmacologically dilated pupil (eg. by atropine) which will not constrict. There may be an associated loss of tendon reflexes, particularly the ankle jerks, but there is almost never any associated systemic disease. Over a period of years, the condition is likely to become bilateral and the initially dilated pupil will gradually reduce in size. However, its poor reaction to light will continue. 2.- Marcus Gunn Pupil-paradoxical dilatation of pupils in swinging flashlight test,seen in retinal detachment,optic neuritis etc. 3.Pupils in pontine lesions-; pontine lesions cause miosis but normal light response. pin-point pupils following pontine haemorrhage; 4.central diencephalic herniation causes fixed dilated pupils: 5.Argyll Robertson Pupil- Accomodation Reflex Present(ARP-mnemonic). 6.Pupil in Uncal herniation- findings include ipsilateral pupillary dilation, loss of light reflex, and ptosis due to compression of cranial nerve III. -Often develops into Korsakof's even when treated -It has a mortality rate of 70 % to 80 % if untreated -The treatment is thiamine 100 mg. PO daily for 5 days -The symptoms include Nistagmus ataxia and opisthotonos -None of the above * 2. Which is the most serious complication of the supra condilar fracture of the Humerus ? -A compartment syndrome of forearm -Failure to heal -Healing in a non anatomic position -Injury into the median nerve. -Permanent restriction of the Elbow motion. 3. Which one of the following descriptors of a diagnostic test is Influenced by the prevalence of the disease being tested for : -Specificity -Sensitivity -Accuracy -Positive predictive value -Reliability 4. A 43 years M admitted for Emergency Gastrectomey present confusion on the 3rd post-Op day. complains of lack of sleep due to cockroaches on the ceiling he is

http://www.usmleforum.com/files/forum/2007/1/193828.php[11/7/2011 2:10:26 PM]

Remembered questions for STEP 1 - USMLE Forum noted to be flashed & tremelus by the nurses during the day .. the most likely problem is : -Post-Op Electrolites Inbalance -Paranoid Schizophrenia -Depression Psychosis -Delerium Tremons -Anoxic brain system 5. Pt. with known type II DM. treated with Clorpropamide present to ED in comma .. blood sugar = 1 mmol / L. .. you give the Pt 1 ampule of D50W & the Pt wakes up promptley . what is the next appropriate management : -Give another ampule then discharge. -Give 2 ampules then discharge. -Give another ampule then observe the pt for 6 h. in the ED. -Give one more ampule , admit the Pt. & start IV. Glucose Infusion with frequent ongoing glucose measurement 6. Pregnant 36 weeks with vaginal watery discharge , management : -do sterile speculum exam 7. Barbiturate withdrawal = convulsion 8. Retrovarted uterus complain = asymptomatic 9. G5 P4 post-delivery, postpartum hemorrhage, most likely cause is : -Uterine atony 10. Anal skin tag in child associated with , -chronic Anal Fissure 11. Child with abdominal pain attacks , drowning his legs up , mucouid bloody stool. Diagnosis ? -Intussusception 12. 1st sign of foot gangrene is = rest pain 13. Pt. M pain in both lower limbs with week popletial artery pulsation , management ? transvertebral angiogram 14. basket ball player averted his ankle joint during jumping at match , on Examination Pain with Increased Various range. Management ? -Repair ligament surgery -Below knee cast 15. clean wound cut with laceration & incomplete section of nerve , management ? -Suture of wound Immediately -Leave the wound open

http://www.usmleforum.com/files/forum/2007/1/193828.php[11/7/2011 2:10:26 PM]

Remembered questions for STEP 1 - USMLE Forum 16. pain on walking increase in 2nd & 3rd metatarsal bone of forefoot. Diagnosis ? -Stress fracture -Sesamoid bone 17. Pt. 68 years going to elective surgery , he has 4 mo. history of chest pain & got same attack at night before the day of surgery , what do u do ? -Lignocaine before surgery -Trinatreate during surgery -Send him for ICU -Postpone the operation & check function of the coronaries 18. collage student F 20 years presented with lower abdominal pain , PE bilateral lower abdominal tenderness , febrile Vaginal exam : tender with cervical mobilization , pregnant test (-). Diagnosis ? -Acute salpengitis -Ectopic pregnancy -UTI 19. vaginal bleeding , stooped at the day of admission , pregnancy test is (+) US shows empty uterus & left adnexal mass. Diagnosis ? -Ectopic pregnancy 20. young Pt with history of non productive cough but clinically well .. C-X ray shows bilateral basal Infiltration .. Diagnosis. ? -Mycoplasma Pneumonia 21. Pt with Ulcer in sole of the foot , at 2nd & 3rd metatarsal joint X Ray necrotic & destroyed part of 2nd & 3rd metatarsal bone .. what to do ? -Radical debridment -Above knee amputation -Daily sterile dressing -Oral antibiotic 22. Problem which cause most work days = back pain 23. Nicotinic skin patchy is contraindicated in: -Pregnancy -CVA -Ischemic Heart disease -Alcoholic NB. (not sure) 24. Psychotherapy is superior to medication in: -Schizophrenia

http://www.usmleforum.com/files/forum/2007/1/193828.php[11/7/2011 2:10:26 PM]

Remembered questions for STEP 1 - USMLE Forum -Bipolar disorder -Alcoholic withdrawal -Dysthimia 25. mother worried about her child because of history of myopathy in family .. what Investigation to be done 1st ? -CPK -Muscle biopsy -Nerve biopsy -EMG 26. Pt on Lithium therapy became weak, lethargic, Intolerant to heat. what Investigation must be done ? -Lithium level -BP -Thyroid Function test 27. surgical nurse 25 years old is concerned that she is loosing her mind, for the past 6 months she is been preoccupied with contamination, on his ward she has been not touching any patient ,door knobs ...etc. She was also washing her hand excessively, which of the following treatments will help in reducing her preoccupation and hand washing ? -Fluxetine -Lorazepam -Perphenazine -Insight-oriented psychotherapy -Nifedipine 28. M 50 years with Prostatic Cancer with bone metastasis. Treatment ? radical prostatectomy -Radiotherapy -Hormonal therapy in the form of total androgenic blockage -IV chemotherapy 29. Pt with family history of urticaria, presented with urticaria ... investigation revealed C1 esterase inhibitor deficiency ... Diagnosis. ? -Hereditary Angioedema 30. thickened upper Lt. lip with mild vesicles ... Diagnosis ? -Urticaria -Angioedema -HSV infection

http://www.usmleforum.com/files/forum/2007/1/193828.php[11/7/2011 2:10:26 PM]

Remembered questions for STEP 1 - USMLE Forum 31. Erithematous Vulva with whitish lesion of Candida of Groin & Satellite lesions .. what is the predisposing disease ? -DM -CA. vulva -Lichen sclerosis 32. Pt with sudden cough & pain in the Rt. side of the chest .. you suspect aspiration of foreign body .. what finding of the C-X ray ? -Hyperlucent Rt. side 33. Defrentiaion between Maxilary & Zigomatic fractures ??????? 34. calculation of the effectiveness of a vaccine Incidence Non Vaccinate - Incidence Vaccinated --------------------------------------------------------------x 100 Incidence Non Vaccinate 35. child chalking during eating .. he became Cyanotic, agitated, with forceful breathing .. 1st action to do is : -Hit him from his back while head downward -Introduce your finger in his mouth 36. 17 years F. at stage 3 Tanner still not menses .. management ? -Examination of the pelvis -Chromosome analysis -Estrogen Progesterone level -Reassurance 37. asthmatic Pt. presented in the ER with Dyspnea , used to inhaled salbutamol with no Improvement.. PE shows dyspnea slit rhonchi at the upper chest & neck ... management. ? -IV. fluid + Aminophine -Coricosteroids Inhaled -Inhaled Salbutamol + IV hydrocortisone + IV Fluid 38. Alfa fetoprotein Increased in -Menengomyelocele -Renal Agenesis -Down Syndrome 39. pt with abnormal pap smear .. what is the next step ? -Colposcopy 40. 60 years F. presented with 5 x 5 cm. adnexal mass .. management. ?

http://www.usmleforum.com/files/forum/2007/1/193828.php[11/7/2011 2:10:26 PM]

Remembered questions for STEP 1 - USMLE Forum -Surgery -Chemotherapy -Radiotherapy 41. Pt. found unconscious on the floor behind the bar, alcoholic, ER agitated semiconscious ... PE shows laceration on the head, all the limbs can be moved, all the others are normal?what is your action? -CT of the head -IV. fluid + Observation -Discharge 42. Pt. fail down from the 2nd floor on his back .. the following will be part of the Initial management EXCEPT : -Cervical collar -IV. fluid -Spine X Ray -Urinary catheter 43. tertiary prevention is -Rehabilitation. 44. in the ER. young Pt treated with Procainamide his BP. decreased to 8060 you tried another time with Procainamide .. his BP. still decreasing .. what to do ? -Benztropine -Saline perfusion -Digitalis -Defibrillation -Intubation NB. the Pt. had Arrhythmia with hypertension (that’s why we gave him procainamide) & because its not decreasing we have to defibrillate him ... 45. F 23 years present with fibroadenoma of the Lt. breast in the lower medial quadrant of the breast all are possible EXCEPT : -Give Estrogen -Give Progesterone -Fine needle aspiration will bring clear liquid -Mammography can show abnormality -1-Child 3years with swelling of scrotum. Testis can be palpated through the swelling Fluctuant, painless. Observed for few months. Translucent. Definitive Management? 2- Question on probability- probability of finding one disease is A and other is B. (Independent). Probability on finding the 2 diseases in one pt. a) AX B

http://www.usmleforum.com/files/forum/2007/1/193828.php[11/7/2011 2:10:26 PM]

Remembered questions for STEP 1 - USMLE Forum b) A+B Full term child. Birth weight 3000. Develops respiratory distress. Xray shows air bronchogram. Diagnosis The growth of the Tumor following cycles 46. F. febrile with tumor in upper Rt. quadrant of the anus .. the mass is painful & red .. what to do ? hot bandages -Cold sitz baths -Surgery -Systemic Antibiotics -Topical Antibiotics 47. child with juvenile poliposis .. possible complication : -Diarrhea -Painless bleeding -Painful bleeding -Malignancy (cancer) -Non of the above 48. a mother brought hear 12 years daughter who present repetitive UTI temp. 38.5 , IVP & cystography are normal .. what is the best test to do : -Urine culture -US & voiding cystography -Blood culture -Cystoscopey -None of the above 49. fibroadenoma of the breast what is false : -The most common benign tumor -Fine needle aspiration bring clear fluid -True solid tumor -Tumor easy to find because encapsulated NB. check it alone 50. F pregnant 28 years .. with nocturnal numbness in hands & forearm which wakes hear up for 3 months .. she also have difficulty grasping objects .. what is the most likely etiology ? -Dermatomiosis -Abruptio placenta -Multiple sclerosis -Carpal Tunnel Syndrome -Hyperventilation syndrome

http://www.usmleforum.com/files/forum/2007/1/193828.php[11/7/2011 2:10:26 PM]

Remembered questions for STEP 1 - USMLE Forum 51. 72 years old with heart failure with high blood pressure treated for long time. He was brought to E R for hypotension 8060 HR 110, what to give ? -Procaine -Lidocaine -Cardioversion -Digitaline -Furosemide 52. Post-op. of Pancreas .. what is the cause of hypovolemic shock ? -Initial necrosis of acute pancreatitis has produced tripsine -Septic shock -Loss of liquids -Side effect of anesthesia NB. not sure 53. 68 years F. with history of Hysterectomy, came to see u because of micturation trouble at effort, no digestive trouble, she has a feeling of a painless mass intravaginaly, she has to try twice in order to void her bladder what is the diagnosis ? -Rectocele -Cystocele -Prolapse (Prosedentia) -Urethral sphincter spasm -Post-surgical stricture 54. young M. feels sudden crack in his calf, what is the best clinical sign to diagnose Achilles tendon rupture ? -Decreased dorsal flexion -Impossibility to walk on the toes -Increased passive dorsiflection of the foot -Squeezing calf does not passively planter flex foot 55. farmer 74 years M. present with bilateral semetrical neaurosensorial defenses , PE is normal . diagnosis ? -Autosclerosis -Professional defenses -Acustic neuroma -Presbiacusis -Circulatory deficit NB. Most common cause of hear losing in elderly is presbiacusis 56. F. 45 years with decreased visual acuity, no pain

http://www.usmleforum.com/files/forum/2007/1/193828.php[11/7/2011 2:10:26 PM]

Remembered questions for STEP 1 - USMLE Forum , no tears. what do u exam first ? -Tonometry -Refraction -Angiography -Neurologic test -Rheumatoid factor screening 57. Burned Infant 2nd degree in the upper limb, what do u do ? -Debrid & skin graft. -Debrid & bandage -Debrid & topic cream -Clean the wound & systemic antibiotic -Local care only 58. Cyclist attacked by bee comes to emergency with hypotension 8060 , Heart rate 115 .. what do u do first ? -Antihistamine H1 & H2 -Saline perfusion -Epinephrine Injection -Intubation -IV. steroid NB. we also give it by subcutaneous or IM. & in children IV or Endotracheal tube 59. child 5 years bitten by the neighbors dog .. the dog received all the vaccines. what do u do ? -Observe the dog 10 days & anti rabbi serum -Observe the dog & anti rabbis serum & vaccine -Kill the dog -Vaccinate the kid -Observe the dog 60. F. 28 years present with chronic rhinorrhea , in the exam you found mucousal nasal atrophy .. diagnosis ? -Sinusitis -Cocaine intake -Nasal poliposis -Allergic rhinitis 61. F. 45 years back from a plane travel , complains vertigo, tenitus, moderate hearing loss, BP is 160110 .. Diagnosis ? -Hypertensive crise -Miners disease

http://www.usmleforum.com/files/forum/2007/1/193828.php[11/7/2011 2:10:26 PM]

Remembered questions for STEP 1 - USMLE Forum -Migraine -Acustic neuroma -Barotaruma 62. what is the most common cause of lung abscess in post-Op pt. -Bacterial discrimination from operative site -Atelectasia -Pneumonia man with prior history of recurrent calcium kidney stones; is hospitalized and passes another stone which was found to be composed of calcium; what do you do as the next step in treatment? -choices were -diuretics, advising the person with diet, I think, stuff like that. One of the choices was administering Furosemide, which is what I chose ……pic of colonoscopy specimen next to diagram of colon w/ rectum circled; mentions neutrophils found in crypts;--but basically asks for dx; I chose ulcerative colitis (crohn’s was also a choice) ……….pt presents w/ pain decreasing w/ meals; how would test for causative organism? I put urease breath test ………..pt comes in after eating at chinese restaurant feeling nauseous, dizzy, and some other stuff; ques asks what could have caused this, and lists bunch of amino acids; I chose glutamate, thinking reaction to MSG ………..fetus is found to have bilateral renal agenesis; what other features would be associated w/ this? I narrowed it down to anencephaly and pulmonary hypoplasia; ans is pulmonary hypoplasia, which I unfortunately didn’t put… argh ……..man presents w/ painful big toe; negative birefringent crystals found; was treated with diuretics for htn, I think, before this; painful big toe could’ve been prevented how? I put by administering something that inhibits xanthine oxidase (sorry, can’t remember the other choices) ………..elderly lady presents w/ increasing difficulty getting up from seated position; find stiffness in range of motion in all directions w/upper extremities, or something like that; and then I think there was an intention tremor, but I’m not sure… basically, the only 2 choices that made some sense to me were Parkinson’s and ALS; I put ALS (unfortunately, I can’t remember if there was any mention of cognitive deficits, but I don’t think there were… sorry again!!) ………..CT scan of head showing either crescent-shaped hemorrhage or biconcave disc hemorrhage; just know that crescentshaped = subdural hematoma, and biconcave disc = epidural hematoma, and that epidural will have symptoms very quickly… ……….Problems w/ UV-light are caused by deficiencies in DNA repair (right?) …………Pt w/ myasthenia gravis; what’s the effect on the postsynaptic neuron? I put something like, decreased EPP, or something like that; other choices dealt w/ amplitude in presynaptic/ postsynaptic neuron ……….Deficits from damage to cerebellar vermis vs. hemispheres ……Lesion is found on ventral root at L3; deficits would be seen where? At L3, below L3, above L3… I think I overanalyzed this one, â €˜cause I put below L3… I’m thinking it was at L3 ……….Ques about differentiating pancreatic cancer from cirrhosisâ €¦ ……..Pt has one kidney removed to donate to relative for transplant; 6 weeks later, what will GFR be? Choices included decreased by 10%, 20%, etc., or no change. ………..Mother finds out she must deliver fetus before term; sphingomyelin:lecithin ratio is low; administering what will help baby? Ans was glucocorticoids, I think

http://www.usmleforum.com/files/forum/2007/1/193828.php[11/7/2011 2:10:26 PM]

Remembered questions for STEP 1 - USMLE Forum ……..Couple are both carriers of hemoglobinopathy; hydrops fetalis was found in baby; what will RBC morphology be like? I put microcytic ………Pt. found to have positive osmotic fragility test; may have listed some other stuff, but I put dx as hereditary shperocytosis …………Pic of foot, w/ tendon torn that attached to lateral aspect of pinky toe; which muscle was affected? …….x-ray of hand w/ sharp object piercing between 4th and 5th fingers; which artery was affected? …….pic of an Ig asking which region is the constant region and variable region… sorry …..young child presents w/ recurrent infections w/ s.aureus and aspergillus; negative nitroblue tetrazolium test; had all immunity injections; norm antibody levels; this is deficiency of what? I think I put oxidase deficiency; …..in fetal circulation, travelling from umbilical vein to heart, through which do you need pass? I put ductus venosus ……in pt, observe medial border of scapula protruding when patient puts arms against wall; which nerve is damaged? Long thoracic nerve ……pt w/ inability to use left arm; show pic of humeral head w/ tendon retracted from top of head; which muscle’s tendon was this? choices included suprasinatus, infraspinatus, teres minor, teres major, and biceps; I think the answer’s supraspinatus ……damage to recurrent left(?) laryngeal nerve will lead to weakness in what? .......….hiatal hernia might also affect which structure passing through the diaphragm nearby? Thoracic duct and azygous veins were choices; can’t remember what I chose ……..Pt presents w/ infiltrates in lungs; able to tell from exam when listening to lungs in left (or right?) mid-axillary line; which lobes are affected? …..Woman undergoes c-section; after procedure, can’t stop profuse bleeding; which artery may have been severed? …..Pt has poor repetition, good comprehension, and I think difficulty recognizing fingers, or something like that; qs asked for which area of brain may have been injured; looking at 1st Aid, I’m thinking arcuate fasciculus, ….Ques involving internuclear ophthalmoplegia – woman has difficulty w/ lateral gaze and can’t converge eye that has difficulty w/ lateral gaze; which muscle was affected? Ans related to SO4, LR6, Couldn’t think of that mnemonic when I was taking the exam …..Pt undergoes surgery and falls into coma, w/ chance of normal survival very slim; no advance directives; upon speaking w/ family further, physician finds out that pt did not want to live in persistent vegetative state; doc then terminates life support; what kind of decision did physician make? Choices included: decision from limited resources, and some other one ……Obese woman is chronically tired; husband reports that she is very loud snorer; can’t remember rest of the ques, but something to do w/ sleep apnea; may have been dx ques ….a ques involving narcolepsy and what kind of hallucinations are associated w/ the condition ….Ques involving delirium – after surgery, patient is in and out of consciousness, hallucinating, and having illusions …..Child is constantly creating trouble in class; bullies other kids sometimes; will be able to be sustained in task where he has interest, though; dx? I think I put conduct disorder, but I’m not sure if thatâ €™s the answer …….Eating disorder ques… can’t remember if it was anorexia or bulemia, but the question was asking for what could be a condition in the patient a lot further down the line in the future… I think I put joint problems or something, but I can’t remember what the other choices were… I just had no idea …….Whenever child cries, mother gives the child a treat to quiet him down; what kind of reinforcement is this? (or something like that)

http://www.usmleforum.com/files/forum/2007/1/193828.php[11/7/2011 2:10:26 PM]

Remembered questions for STEP 1 - USMLE Forum orotic acid which pathway albright synd... whr is the defect? turner synd how to concieve.....hcg...ivf mutation in hyrophobic signal sequence....whr will be products accumulate....rer endosome ant spinal artery occlusion arnold chiari thrombosis in MGN ...wat coz dentine dyplasia whr protien accumulate genetic imprinting reyes synd patho in brain.....damage to cellular mito. praider willi thynic tumor pic of lacerated wt type of wound....tearing wound. pleotropy moa of azathiioprin mom o 'neg,baby o' pos, father's blood type?a 'pos or o ' neg. ---A pt with Turner synd. Ask you about getting pregnant. What do you advice her? Oocyte transfer, IVF ---Patho slide with given vignette Hypertrophic Cardiomyopathy MCC of death? Arrhythmia --- Vignette : a young woman with high fever, septic condition INR increased fibrinogen decreased, etc Answer : DIC ---Two TTP question on different block --- wWd dhat do you give him after desmopressin: Cryoprecipitate ---Warfarin overdose ---Squamous cancer of lung with hypercalcemia: Ectopic PTH ---PT with numbness and tingling on dorsum of foot ankle reflex decreased, where is the lesion: L5S1 disc compression ---A pt with Turner synd. Ask you about getting pregnant. What do you advice her? Oocyte transfer, IVF ---Patho slide with given vignette Hypertrophic Cardiomyopathy MCC of death? Arrhythmia --- Vignette : a young woman with high fever, septic condition INR increased fibrinogen decreased, etc Answer : DIC ---Two TTP question on different block --- wWd dhat do you give him after desmopressin: Cryoprecipitate ---Warfarin overdose ---Squamous cancer of lung with hypercalcemia: Ectopic PTH ---PT with numbness and tingling on dorsum of foot ankle reflex decreased, where is the lesion: L5S1 disc compression pay attention to HIV drugs 1- 21 alpha-hyroxylase def 2- prolactinemia, lactation…pt. not pregnant 3- pt. gone 3 months without menstruation…Dx? Pregnant?....? 4- Pt. with Turner Syndrome, streak….45XO 5- Erecrtion problems…what venous sys affected? 6- Pt. with Infertility problem. 7- Huntington, trinucleated repeat 8- Non-communicating Hydrocephalous. Gross pic shows block between 3rd and 4th ventricle 9- Remember where spinal pathways crosses….medbrain? 10- Iron Def. pt. c hypochromic microcytic, decrease iron stores in bone marrow 11- hyperpigmented neutrophil(pic) pt with neurologic def. …B12 def 12- Pt. on vegi diet….? B12 def. 13- Pt. Sleepy during the day. Dx? Tx? …Sleep Apnea ...had a lot of ct scans and Mri.....but one thing I will emphasize that basic is critically important. e.g ...gen pharm. and gen patho is must. ..... qs about comp inhibitor with graph lots of lines crossing and had to identify the comp inhib. ...Q about efficacy of drugs on graph. ...q about bony defect on the back of newborn with tuft of hair on it=spina bifida ...again q about meningomylocele asking contents. ...hirshprung disease whts the cause of disease=failure of neural crest cell to migrate ...q about 40 yr old with 20 yr H/O type2 diabetes tingling and sensory

http://www.usmleforum.com/files/forum/2007/1/193828.php[11/7/2011 2:10:26 PM]

Remembered questions for STEP 1 - USMLE Forum prob in limbs=pripheral neuropath. ...young women with sudden knee joint swelling= trauma( septic arthritis was also given but it was asking sudden) ...occulomotor lesion in at least three qs. ...midddle cerebral artery lesion ...MRI of sp cord =subacute combined degeneration ...pt has accident and lost conciousness after 18 hrs. type of hamorrhage was asked. ...q about dysphagia even for liquids =achalasia. ...lots of neuroanatomy ...men1 =ret gene ...diabetic keto acidosis= what happen to oxygen dissociation curve, had to identify on graph ....acid base balanece graph in person with resp problem. ...tension pneumothorax ...signs and sypmtomps were given with Xray ...basal cell ca ....best treatment option? ...granulation tissue.... what day of wound healing ...a lot of immunology......genetics with konckout mice ...every block about 3 doc pt relation...... ...Waldenstorm gglobulinemia ...gfr and rpf with efferent art const ...thyriod was asked the most in endo X-ray shown there is an artery on the right side and asked about it. peronial nerve easy. showed all the bracial plexues and asked to identify the nerve that getes effeted if there is a humurs neck is fratured Vertical deplopia cirlcel of willis diagram and asked which artery effected if the person legs are affected major drift in influeneza and asked how it occured A child has dermatitis since he started new school adn he rides his bikes to school and also plays in football feild after school and described as contact dermatitis, and asked i 1. caused becos he sweat when he is rides the bike 2. poison ivy form play gound 3. on the way to school becuse of sun i think it was A man gets into an accident he had his 6 years old sun wiht him in the car, boy got contution on his head a some other injuries asked , was the boy on front seat was he wearign a seat belt some ansewers liek that You and a anesthalogist , something about gestric surgery and anestahogist said somethign about man being fat do you talk to him in privat talk to him when witness presant do nothing take him to the commite Pku the boy is put on a diet and phenylalanin is under control what else should to test him for. Lysch Nyhan: describe 7 years old boy, self mutilation, mental retardaion, what some enzyme si missing and wht will accumulat, Uric acid Mcardles disease, no increas in lactic acid when the girls excersises: myophorylase. A woman has rohmatoid arthritis she is given perdenosin, then you inc the does started to feel really weak, predinosin side effect chronic rohmatoid syndorm some other choices Described a man he had a strok showed 2 picture then he started to see some changes did say much about where and said one pic from 9 month ago one from now and said what part of body t effect. and where Seborrheic keratosis: "stuck on" appearance of this verrucoid appearing, pigmented lesion. A woman taking ACE inhibitor has a dry cough which other medication would you give heir Losartan .A 55y/o man came in to you clinic, he had blood in his unrine, his prostate is enlarged and firm, and there is a mass above his left

http://www.usmleforum.com/files/forum/2007/1/193828.php[11/7/2011 2:10:26 PM]

Remembered questions for STEP 1 - USMLE Forum kidney. Smoker fro 34 years, Whats your diag… 1. pyelonephritis 2.prostitis 3. renal cell Ca 35, years old came in complaing that her shoes don’t fit her anymore, her jaw widened,a nd there is gap in her teeth, what hormoned is responsible for this, 1. somatostatin A 2. prolactin 3, eastrogen 4. GH 5. LH 3. A 34 years old and her husband come in to the clinic, they want to have a kid, the woman cannt get pregnant, there is nothing wrong with the woman and her husand all the level of hormone are normal, what cannt she get preg… 1. I piked a choice where it said somtheing like she is PID in the past 4. A nurse came in with palpitation, complain god anxiety, heat intolerance, and she admitst that she is taking some pills to lose weight, and values of TSH, T4 and T3 and given, you dia 1. Hasimoto 2. factitious disorder 3. primary hyperthyroidis 3. Hypothyridism 5. A woman pre described having polyhydamnios whats risk in kid? 1. Renal agensis 2. Esophageal atresisa somother choieces 6. A man came in with H. pylori you gave him, proton blocker what is the PH in of stomach something like that A woman came in with bloody diarrhea for 5 days, no travel history, she just got a dog and the dog like to lick her face organism causeign the diarrhea. 1. E.coli 2.campylobacter 3. Yersinia pestis A man who came from berzil, had PPD negative then he has meseals a years later his PPD was positive. MOA how it was positive How does N. gonorrea cause the inf. 1. Endotoxin 2. capsule 3. pilus 4. flagellum 5. spores Described Actinoyces : it was easy How do you differentiate spcies with in Streptocouccus something like that. 1. coagulase 2. catalase 3. hemolysis A older man came in with pneumonia , he recived tetnus vaccine 5 years ago.. Which vaccine should he be given Osteoyelistis the described in a kid what sickle cells Over expression of TNF- Alpha what will it cuuse in a mouse MAC formation Dec macroghages and IL-2 Dec T lymphocytes A child got into accident and needs blood transfusion, he is has Ig A def which of the blood is contraindicated in him A normal persons Someone with Ig A def And somother choices I don’t remember MOA how methicillin causes Interstitial nephritis A man taking procanamide get Lupus but it was a long Q give all sort

http://www.usmleforum.com/files/forum/2007/1/193828.php[11/7/2011 2:10:26 PM]

Remembered questions for STEP 1 - USMLE Forum of inf. How does a organism becomes resistant to Amoxicillin, Cephlosprins and not Aminoglycosisdes A 18 years old develops rash all over his face, after he started a new acne medication, he has a sore throat and is also taking Medicaiton for that, then they gave some names of AB he is taking , and one of them was Tetracyclines so I picked that A man has Aid and is infected with Pneumocytis carini and he wants to take something for proflaxis, 1. Ramfipin 2. TMP-SMx A man just had PPD positive the drug with wich MOA should he be give: I picked dec synthesis of mycolic acid A girls just developed dertophytoses, she is started on Terbinafin: MOA of this drug 1. Inhibit fungal enzyme squalene epoxides A man with aids developes CMV he is given a medication: MAO of this medicartion A man who have had a depression problem for past 10 years and tried 2 kill himself on two occasions, is broght by his neighbors because he cannt sleep he think his neighbors dog is in the you clinic and is going to bite you, what medication would you give him, Atypical antipsychotic, pluexetine, amitriptyline, benzodiazephines A boy is havig sezures and described it was partial complax A man come in he is being treated for depression: Name of medication that is causeing urinary retention, tachycardia etc. I put amitriptyline. A man got in to an accident broke a lot of rib bone and leg bones he give pain medication (morphine, and lorazepam) and after 30 min he is still in pain 1. Medicaiton is has short half life 2. increasead intolerance som other choices A man is on clopidogrel is should sid effect: Neuropenia A man is under chemo why is allopurinole added to his regmine: inhibits uric acid formation. A girl comes in with inflammation of medium-sized muscular arteries, involving renal and visceral, she is also positive for HBV, next step she is in pain, cyclophosphamide and coricosteriods weren’t in choirces, it was all the antinflammatories A man’s hand were shown and Raynaud’s diseasse was described: what else he has; PSS Described Kayser-Fleischer ring: and asked where is copper being accumulated Tension Pnumpthorax explained and gave 5 xray and asked to pick one Epidual hemaotoma Pic of diardia lambila shown and aked what it cause: bloody diarrhea, malabsorption Guys I Had atlead 15 Q just on 2nd messenger: there is a page in First AID G-protein linked 2nd messengers. Know it well. There were one line Q and confusing Described a child with maple syrup urine disease, and asked what Vitmin should be given to him. Also child with led poisoning what is accumulated in urine. 3 quesiton on Vit B12 asked what is in urin if def, A boy with I dotn remember with what but if his spleen is remobved he will be susceptible to what kind of infections: Transduciton descried

http://www.usmleforum.com/files/forum/2007/1/193828.php[11/7/2011 2:10:26 PM]

Remembered questions for STEP 1 - USMLE Forum A 25 y/0 girl whith disseminated diseas with the constitutional symptoms rash on palms and soles. Described VDRL, FTA-ABS medthod and asked about which one is positive I ma not sure A man is acting strange, otherwise healthyand some an organism is found in his temporal lob: 2 Q on influenz virus and vaccine What is nevever normal in a person flora: staphylococcuss, viridian, strep, TB, Ecoli A12 diabetic boy is not taking his not taking is medication regulary : 1, involve in actively and dicuss with mother, some choices like that very confusing A 34 y/0 college of you has strep throht, he asked u to give him some sample AB if you can because he is really busy and cannt see his physican. You response should be 1. “do you think its strep throat†2. I m sorry you should see your physican 3. I will give you a AB until you see you physian 4. I will give penicillin for 10 day How does cholestryramines work A long Q about a man with high cholestrole and asked about which medication acts also like an anoxident Described Torsades de pointes and asked about the medication caused it Sotalol was in the choices Also gave Arachidonic acid sturcuter and asked where does asprin workd very werried Q A man has asthma, diabetes and hypertention: what is contrindicatied Albutrol Propnolol Cromolyn Throphylline How does Tomoxifens work they didn’t give he name but descriebed it Why is mesna given along with cyclophosphamide Renal cell Ca slid is hown and asked for risk factor: I put smoking, it was a har Q because they just showed the pic and said what cause this the slid was form Goljan so I knew Guy I had 10 Q from breast ca I coundted all of them all slid were shown I didn’t really study breast ca and all of those are in goljan slides I just saw and know as goljan the location very hard Q and also asked about prognosis Alcholic with pancreatic adenocarcinoma: pain readiating ot back Shown x-rya and asked which of these is x-ray of a person with restrictive lung disaes Also 1 Q about Emphysema described it. A q about a girl has high ammonia concentration, OTC def Interssusception: which seg most involved Muliple myeloma: guys you had to know it by looking at slide and said what would you see in urine of this person: Hurschsprung’s disease what would you see in the intestinal biopsy: missing ganglion Described Gout and asked what kind of crystal would you c Hemophila A confusing Q DIC 2 Q Hyperaldosterinism in a child it was very very confusing Q Addision Thymus missing what other sturtuer would be missing in this person PTH is inc and phosprus dec y Woman has polycystic kidney diease she at most risk of which cancer: Ovarian, cervical, someother choices Hydatiformole in a woman was found 6 months ago now she presents with, basically they described coriocarcinoma Non communicating hydorceplus: showed tumer blocking 3rd venticale Which hormone is involved directly in formation of polycycyctic ovarian syndrome: LH, FSH, testestron, estrogen. Ectopic pregnany described Homeboxe gene abnormalties

http://www.usmleforum.com/files/forum/2007/1/193828.php[11/7/2011 2:10:26 PM]

Remembered questions for STEP 1 - USMLE Forum Something wrong with Androgen-bindig proteins problem with what: sertoli cell, leydign cells, testosterone, Co2 transport in tissues: what form is it in peripheral tissue: Bicarbonate A Q about inc in Vita min D and what happens to ca, pth, phosphate Ricket described in a child asked what caued it: Ans another Q about Ricket what kind of bone would you C. One Q how does bone get nutrients Same Q form UW, if you constrict efferent arterioles what happens to RPF, GFR, FF Also q form UW 3000kcal and 30 from protein: 200 Q with a women with diabeties has renal problem what medication should she be give to slow progression: ACE inhibitors ans Q about baroreceptors: dotn remember exactly PDA murmur described An other one about senile purpura in old man not palpable: described senile purpura A child has meningomycelocele what else would you C Show 5 slide and asked which slid is sertoli cells Adenoma polyp showed and asked for diag: Described a man with burkitts lymphoma: asked what when worng: Translocation I got this wrong I was thinking apoptosis I donno b I houth it was follicular Def of pyruvate kinase what would you c in urine Hurler: what is accumulating Petigery of Muscular dystrophy: very confusing ans Skew X promble I put everything else didn tmake sense When insuling inc what other hormone inc I dotn remember the Q A diabetic pt. comes in he excersies he is taking care what is complient what else shold you tell him : I put check you feet everyday Sarcodoiss Q very long and confusing Colon cancer APC Billary cirrhosis: in a woman A man with hepatocellular ca inc PT time what would you C: AFB 1. what kind of receptor on bronchi? beta adrenergic, Muscurinic receptor 2. bicornu uterus is due to failure of fusion mullerian ducts 3. chornic pain syndrome , where is the defect? 4.60h/o head injury 10 year back, recurrent rhinorrhea, what's finding on CSF? If no infection, normal CSF 5.h/o radiation exposure in neck........what cancer would develope Papillary carcinoma of thyroid 6.what make unconjugate bilirubin water soluble-conjugate? 7. something like that in my exam ....... as part of reserch by medical student , cardiac cell has Ap of + 60 mv firt time and + 30mv second time .......... why it's decrease in second time ...... a. student add more Na+ outside cell.... b. student remove the Na+ from outside cell..... c. student add more K+ inside cell d. student remove K+from inside cell 8. withdrawal from which of the ff substances is most likely to produce a life threatening syndrome in a person dependent on that substance? A amphetamine B cocaine C heroin D Methyphenidate E secobarbital 9. clinical trials have suggested that retinoic acid can induce remission in pxs with acute promyelocytic leukemia/such remission is related to the abilty of retinoic acid to promote which of the ff A differentiation of leukemic cells

http://www.usmleforum.com/files/forum/2007/1/193828.php[11/7/2011 2:10:26 PM]

Remembered questions for STEP 1 - USMLE Forum B diffrentioaation of monocytes into macrophages C generation of cytotoxic T lymphoctes D Production of interferon E repair of DNA 10. 42yr old man with pnuemoccal pnuemonia has acute fibrinous pleuritis.which of the ff proteolytic neymes is required to eliminate tthe exudate and restore normal anatomy? a collagenases b stromeolysin c plasmin d thrombin e trysin 1.Tx of dermatophytosis..... 2.Malaria Q...like an asian immigrant, with remittant fever, chills... 3.Anemia Blood picture..meaning cell counts & all parameters...like Mcv,Hct,Tibc... 4.An interesting Xray which actually looked like Situs inversus...but probably wud have been taken with pt in a prone position..ie on his tummy!! 5.role of GAB A in psy Acellular lesion - diabetic nephropathy Specific site of AG toxicity ret gene association - which cancer - as a part of a vignette recycling of Pyruvate - lactate - Cori cycle - enzyme inhibition what happens new kind of osmotic diuretic --blah blah - just plugged in mannitol and I was ok - asked for s/e thiazide and digitalis - s/e pic of splenic infarct corticosteroids and adhesion molecule synthesis in the form of graphs. MOR for aminoglycosides - case scenario Serum Na - Legionella alpha 1-6 - debranching enzyme deficiency - shrouded in a scenario where there was labelled substrate esophageal manometry of scleroderma S100 antigen - melanoma pic of basal cell carcinoma - risk factor in women glyburide s/e S-S bond important in nephrin beta MSH - skin pigmentation - case scenario clos botulinum, Werdnig hoffman - comparison scenario 1. po260 pco2 55, Hb 15 what's the p50........ 2. MOA ATP on allosteric enzyme........... 3. what kind of receptor on bronchi........ 4.h/o polychythemia , what,s change on red pulp of spleen... 5.MOA of superantigen on septic shock....... 6. mutation on CAP .......... 7. length of mRNA is determine by what ............ 8 . unlar nerve injury. 9.middle ligament calcification on x-ray. 10.cervical ribs on x- ray...... 11.common perioneal nerve injury...... 12. lekage of amniotic fluid case potter' s syndrome same as bilateral renal aplasia........ 13. volvulus........ 11 colon cancer ........ 12 bicornu uterus is due to ......... 13. h/o recurrent gonorrhea what the complication after 10 year ........ 14.nucleus of teste on fig ........... 15. medial lemniscus on fig........ 16 find corticospinal tract on medulla....... 17. chornic pain syndrome , where is the defect on CT.... 18 . epidural hematoma which arteris...CT 19 lateral rectus muscle on saggital view on MRI.......... 20.L5_ S1 herniation....... 21. disc degeneration......... 22. h/o child abuse and child die ...... whtat's the pathology on spinal cord............ 23 h/ o parkinson , where is the lewys bodies located on fig.......

http://www.usmleforum.com/files/forum/2007/1/193828.php[11/7/2011 2:10:26 PM]

Remembered questions for STEP 1 - USMLE Forum 24. XII nerve injury........what's the finding........ 25. 4,11 translocation......... i think it's AML........ 26. pericardial fluid where shoud we put needle to drian fluid........ 27.11,22 translocation , what;s the defect on heart...... 28 mitral stenosis case...... 29 aortic stenosis case ....... 30. singla S2 heart sound , where is located on graph ........ 31. ventricular tachycardia on ECG....... 32. cerival biospy .....cell have high nnuclear/ cytoplasm ratio but do not invade the basal membrane ........CIN. 33.separation of chromatid which phase .... 34. benifit of circumcission........... 35 persistant vatilline duct......... 36. zero orderkinetics....... 37. h/ of taking digoxin........what's chage on ECG........ 38.B27 relation........ 39.h/ o COPD........right shift curve in graph........is due to increase......... co2 ......... 2 3BPG............ 40.budd- chiari syndrome........ 41. case of endrometrosis. 42. HTN patient die suddenly, where is the lesion on brian...... 43.case of fibroadenoma brest......... 44. case of brest abscess........what's the common bug....... 45.mutation on TATA box............ 46. anticodon was given........ write the codon..... 47.digoxin toxicity in old age is due to...... 48.MOA of amphotericin B......... 49. MOA of vinblastin...... 50. S/e of bleomycin........ 51.vit that given with methotraxate is....... 52.compatative inhabitor on graph..... 53.s/e of gancylovir 54 s/ e of AZT...... 55. MOA of anstrazole...... 56.MOA of sulfonylurea...... 57.neutrophil migration is determine by what ........ 58. s/ e of malaria drugs........ 59.case of aseptic meningitis what's the finding on CSF......... 60h/o head injury 10 year back, recurrent rhinorrhea, what's finding on CSF...... 61.gulf player weired mole on lower leg.......... what's the risk factor ... 62.case of neurofibromatosis........ 63.clean wound on fig............ what's the healing process.... 64. h/o alcohol taking ......patient is hypoglycemic 50% dextrose is given but patient still vomitting ..........what' s the next step of management .......... 65. what's effect of alcohol on gluconeogenesisi. 66.celiac sprue , duodenal biopsy done ........what's the other lab finding........... antibody to what..... 67. case of rota virus......... 68. case of measles ....... 69. case of cerebral malaria........ 70. case of streptococcus pyogne ......PSGN...... 71. case of s.typhi......... 72.case of meningitis ........ 72 . case of vWF defeciency....... 73.case of follicular lymphoma......... 74.case of AML....... 75. h/o radiation exposure in neck........what cancer would develope.......... 76. embryo derivative of thymus which pouch..... 77. h/o autoimmuno disease ....remove the thymus..... 78. h/o die in utero in 22 weeks ..........what's defect on cytogenetics........they give placenta picture ........ 79. case of turner syndrome....... 80. prevalance is 1/ 39000 what's the heterogygote frequency of child ....... 81. weired pedegree .......i still don't know what they asking ...... 82. epithelial lining of ureter........ 83. mutation HMCII......... 84. Hb 5, polysegmented neutrophil,patient is dyspnea.......what's the next step of management ....... give boold or vit B12........ 85. Hb7 / microcytic anemia ...... what's the next step of management .......measure ferritin or give iron.....

http://www.usmleforum.com/files/forum/2007/1/193828.php[11/7/2011 2:10:26 PM]

Remembered questions for STEP 1 - USMLE Forum 86.h/o poor nutration , fatty liver what 's the case ....... 87. what make unconjugate bilirubin water soluble..... 88.stone on ureter what's effect on GFR...... 89.left renal a. stenosis....what's effect on renin level......... 90 blcok the PTH recepter on kidney what's effect on PO4+, ca absorption on GI/ 1,25 dihydrocholicalciferol, . 91. hyperthyrodism........what's TSH/T4/T3........ 92. grave's disease .......antibody to what ....... 93. anorexa nervosa what's the FSH/ estrogen level............ 94. estrogen produce by which cell ........... 95. spiderangioma is due to.....what excess on blood...... 96.seminoma , lymph drian in where .... 97. h/ o hernia in early age ........what defect ....... 98. left testicular vein drain in to......... 99. h/ o abortion on 12 week , beta_hCG increase ......what produce beta_ HCG........ 100. MOA of Aspirin...... 101. MOA of iburfen.... 102. conjoined baby......what is the number of placenta / amniotic ....... 103. h/o sudden headache , bood in CSF......... what's the the cause ........ 104. case of temporal arteritis....... 105. pathological finding on PAN....... 106.pathology change after 48 hour in MI........ 107 DOC acute gout....... 107.color of fluid in acute gout ....... 108.NSAID resistance acute gout.........what's the next DOC...... 109.recurrent kindey stone ........ which amino acid should add in diet.... 110.suger burn smell in urine ........ which amin acid metabolism defect ........ 111.what's the case of anemia in lead poisoning...... 112 ????+ glycin -------.> hema 113.h/o insulin producing tumor ,glucose is 20 ..........what's the effect on fasting ............they give whole glycolysis chart ......... 114.warfarin toxicity.........next step of management ......... 115 second messanger on hemorragic shock........ 116. how many glucose need to produce one fatty acid ...... 117.h/o epigastric pain , does not responce to pain medicine ...........what's the next step of management ........ 118. site for peritoneal dialysis....... 119. case of BPH....... 120. case of cor pulmonale ........ 121. size of partical that can remove by mucocilliary mechanism........ 122. h/o pacemaker replacement .........does PR interval always same...... 123.h/o chines, pharyngeal carcinoma...........what's the case ..... tobao or EBV......... 124. pathology change on acute rejection ...... 125. h/ o DM they give both kideny an ask what' s the pathology ..........papillary necrosis....... 126. case of transitionl cell carcinoma........ they give 6 fig of urinary system with different shape and size and ask which one is due to trasitiona cell carcinoma........... 127. case of delirium ....... 128 case of schizhophrenia....... 129. case of displacement ........ 130. case of VSD...... 131. nurse give h/o hematuria and back pain..........there is no assotiation between pain and hemature later she threat to sue to doctor...........choices is facticious /antisocial............. 133. patient came to ur office with his dog...........what's ur responce before he enter the examination room......... 134.h/o diarrhea an vominting what's the MOA .......they give new bug something .................hydrophillia.......... 135. case of H. influenza........ 136. 8 year kid h/ o asthma.......... patient do not want to quite the smoking ............so, what's the next step of advise regarding to smoking......... 140. h/o anurea after riding bicycle (saddle injury ) where is the lesion........ 141. drugs work in dista tubule....... 142. damage of pituitary stalk ........what increse....... 143. antipsychotic drugs work in which receptor .......

http://www.usmleforum.com/files/forum/2007/1/193828.php[11/7/2011 2:10:26 PM]

Remembered questions for STEP 1 - USMLE Forum 144. moa of BUSPIRON........ 145. h/o hemorrhoids............what's the DOC ........ 146.S/o TCA......... 147 h/ o depression after surgery what's the DOC....... 148. case of Osteogenic imperfecta........... 149.case of mycoplasma pneumonia.......... 150. h/ o seizure multipal calcification on CT ...... subcutenious oval shape bump.........so, what's the bug....... 151. 75 year old man lower back pain .......osteoblastic errosion ....what's the case of pain everybody, calm down and pray for peace for the world, Ok. No hate and no dirty words to each other, OK. We are all bothers and sisters having the same dream to become a doc and one day to help others and yourself. Ok, my contribution. more precisely, it is math3's contribution, I put all of her qs together so it will be easier for you to read and to benefit. ------------------------------------------0.1 which one of the follo cause asthma? cockroach spider milliede centipede termites http://www.niehs.nih.gov/airborne/prevent/roach.html cockroaches is the answer 1. visual problem and prolactin???hypothalamus or infundibulum,,,choose infundi Hyperprolactinaemia caused by compression of the pituitary stalk (infundibulum) Hyperprolactinaemia may be caused by either disinhibition (e.g. compression of the pituitary stalk or reduced dopamine levels) or excess production from a prolactinoma (a pituitary gland adenoma tumour). 2. for terminationg pr synthesis,,,,atp,gtp.cgmp,catp for terminatin protein synthesis energy req in the form of atp ? no its gtp The mRNA Signal STOP Codons: UAA, UAG, or UGA, There are no tRNAs that recognize the STOP codons UAA, UAG, or UGA. ----Soluble Protein Release Factors RF1 responds to UAA or UAG RF2 responds to UAA or UGA RF3, a GTPase (like EF-Tu and binds in a similar A-site location) RF1/RF2 interact with RF3-GTP, have a similar shape as EF-Tu-GTP-aatRNA or EF-G, and bind in a similar ribosomal site (A-site). In a manner similar to EF-G, GTP hydrolysis drives the movement of the terminal mRNA codon into the P-site, moving the last tRNA into the E-site and off. At the same time, the polypeptide chain is released after hydrolysis of the tRNA-peptide bond. In eukaryotes, only a single release factor, eRF, is necessary. It recognizes all three STOP codons and interacts with GTP. A mutation resulting in a premature STOP codon is called a nonsense mutation. Elongation consists of three distinct steps to add one amino acid Requires three elongation factors: EF-Tu/EF-Ts and EF-G Requires two GTPs per cycle (4 phospharyl bonds) Occurs many times per polypeptide The elongation cycle is similar in prokaryotes and eukaryotes. Fast: 15-20 amino acids added per second Accurate: 1 mistake every ~10,000 amino acids Termination results in the release of the polypeptide chain Requires one of the three STOP codons: UAA, UAG, or UGA. Requires RF1 or RF2, and RF3 in prokaryotes (eRF in eukaryotes)

http://www.usmleforum.com/files/forum/2007/1/193828.php[11/7/2011 2:10:26 PM]

Remembered questions for STEP 1 - USMLE Forum Requires one GTP Each step of protein synthesis (initiation, elongation and termination) requires GTP 3. in aat defic in emphysema,,,,defect in lower lobe 4. hi guys back from exam,,,,,,,,,,,,,,what to say,,,,it was very tough,,,,,,,,,,,,,15%straight forward,,,,,,less from uw,,,,,,,,,,asked about t -tubles,,,urea into water,,,,and a lot of other stuff in a very confusing way,,,,had barely time to finish completng,,,neuroanatomy was easy,,and also genetics,,,,few pharma,,,,,very confusing doctor-pt relationship The Ca which causes muscle contraction is stored in the sarcoplasmic reticulum ,,,,,,,,,,,,,,,i thought it was t -tubules,,,,,,,,,,,,i choose Sr my exam was tougher than usmle ,,,,,structure of tyrosine,,trachesoesophagel fistule=proximal eso blind,,,,i knew this ,,,a lot of options to make u wrong,,,,,,,,,,,,ca channel=L type,,,i knew this,,,,went wrong on T tubles,,,,dont know about ATP muscle contraction,a lot of confusing options,,,,primary lung tumor in asbettose,asked if it was in bronchi,,bronchioles,aveoli,,etc,,, 5. pain anlog med; NE,5-HT,gaba,ach,,,,,,,,,,,,,,,,,,,,i got wrong on this; is NE=amytriptyline for pain? math3 - 03/21/07 14:48 Gabapentin (brand name: Neurontin®) was initially synthesized to mimic the structure of GABA for the treatment of epilepsy. Nowadays, gabapentin has been widely used as a medication to relieve pain, especially neuropathic pain ,,,,,,,,,,,,,,,,,,,,,,,,,,,,thanks i choose GABA,,,,,,,,,,,,,,,,,,,,,,,,,i thought i waswrong anout pain... yes you are right it is NE... via alpha 2.... could be also analog of glutamine receptors, Mu receptor 6. pulsating abd mass,,,,,,,,,,artherosclerosis or marfan,,,i choose artherosclerosis 7. guys what is 7 methyl guanosine ;; its cap for 5' end of mrna during posttranscriptional processing of mrna

8. glands +chondroid celll????????????, adenoma , adenocarcinoma, , chondroma, chpndrosacroma, teratoma something has both atypical glands +chondroid cells,,,,,,,,,,,,,,,,,i choose teratoma I think questions is basicly saying there are 2 different type of germ tissue you are correct math since rest of them it would be only one origin 9. 36 atp for cellelar respiration of glucose; depends which transport they used malate or G3P malate transport would give you 38 G3P shuttle transport would give you 36 choices were 2,6,14,36 10. anatomic snuff box ,carpel bone? scaphoid n trapezius form the floor 11. increased intracranial pressure,,,,,,,,vit excess??????vit a

http://www.usmleforum.com/files/forum/2007/1/193828.php[11/7/2011 2:10:26 PM]

Remembered questions for STEP 1 - USMLE Forum

12. st corneum clipped off,,,what will deposit,,,i chose keratin; u rite...seems like psoriasis ; what were the choices for the st corneum ques? Cells of the stratum corneum contain keratin, a protein that helps keep the skin hydrated by preventing water evaporation. In addition, these cells can also absorb water, further aiding in hydration and explaining why humans and other animals experience wrinkling of the skin on the fingers and toes (colloquially called "pruning") when immersed in water for prolonged periods 13. which aa deficit in kwashiorkor options were glycine,,glutamine.alanine,aspartate no these were the only 4 options,,,,i was going to and fro on this,,

14. another ,,acute toxicity,parania,,fear, ,,,,,,,,,,,,,,,,,,,,options were pcp,,heroine,,barbiturates,benzo yes its pcp 15. tyrosine precursor of dopamine,,,,,,,,,,,,they gave 4 chemical structure

16. during seizure where is the problem??? premotor ,,,primary motor 4.2. The premotor seizures Seizures from the premotor areas are mainly characterized by tonic and postural phenomena, predominant in upper limbs, with adversion which can be contralateral or ipsilateral and symmetrical or asymmetrical. When there is a propagation to the primary motor cortex, clonic jerks may be associated with the seizure 17. kasmala i can not belive , they ask me the same qs ..........like ........... what happen if u add the ATP during muscle contration .................. a. myosin attached with actin b. myosin separate from actin c.......... d.......... but i chose bbbbbb......... myosin separate from actin ....... for muscle contraction,,,,,,,,,,,,,,,,,there were long choices,,,i chose dephosphyr of myosin when atp is bound,,,,i dont know if i am right??? ??? kashmala ,,,there were no time to differtiate with the choices,,,,,the choices were all long,, 80%of q were to mis lead u,,,or asbettose they told he had mesothelioma and asked where is the primary neoplasm,,,pleura,,bronchi,alveoli ETc 18. what prevent urine going into abdominal cavity,,,is it transversalis fascia??? for myosin there was only phosphorrylation,,,no dephos...i chose phosphory another q on insulin was dephosphorylation dont remember exctaly but something like this, In eukaryotes during m RNA post translational modification some part of rna is removed? what you called it?? a) Exon b) intervening sequence

http://www.usmleforum.com/files/forum/2007/1/193828.php[11/7/2011 2:10:26 PM]

Remembered questions for STEP 1 - USMLE Forum c) dna fragment and one more option 2) This one is one hox, the child is born with six fingers in both hand and his 4th and 5th fingers are fused together, he has no other abonrmalities? what gene problem?? i didnt remeber all option but some of them are 1) hox b9 2) pax c3 and some other options that start with other hox 1/pansystolic murmur(ASD.VSD. PDA) 2/transposition OF GREAT VESSELS(h/o 1day old baby cyanotic...) 3/staright easy qs abt deliruim 4/h/o holo systolic murmur....ans MIT REGURGITATION 5/a baby born and died (mother has h/o oligohydroamnios) ....in autopsy .....kidneyagenesis...what other finding?ans pulmonary hypoplasia. 6/ aortic regurgitation....it was an easy qs. 7/h/o autoimune hemolytic anemia........postive combs test was there.it was straight qs actually 8/h/o 3 pain one sexual prob thou it was big history ......ans somatization disorder 9/a women she always in hurry,worry abt lot of things abt her parents abt kids and ....call her hubb.... that he safely reached his office....ans generalized anxiety disorder 10/ h/o ADHD ASKING ABT MEDICATION....I CHOOSE MEtAMPHATEMNIE. 11/MOA ibuprofen 12/megalablastic anemia....actually it was h/o a patient who was on chemo agent so they were asking abt which vit defciency coz ........so ichoose megalablastic anemia 13/ h/o alcholic ......Rx B1 14/alziemer pat .....describing abt wasting it was long history thou....ans kwashoirkor 15/14 yr old kid ...dose nt want a read ,but he is good in science geography, play games ,he was active drug screen negative,,but he just dont want a read.....there r chosie but i choose 'normal 'coz teen agers,they dont want aread,and other choicees dosent match with history either. 16/ effernt arteriol constricted.....so i choose FF inc GFR inc but renal plasma filo decreease .u can chk in FA the same thing is thr. 17/there is h/o of grandfather ....thr her grand daughter she was giving all the history sho din let granda to ans doc qs ,so wat ur response i choose....tell lady to go out so doc can directly ask qs to the patient. 18/ hypertensive patient....on salt restriction.....he came for follw up...he said doc i don like taste of my food....wat ur response....thr r cpl of reponse but i choose..... give him pocket guide of salt restiction menu.... 19/ h/o immigrant kid KOREA n...cant speek english....translator hired....thr r some h/o coin lesions on the bak of kid...wat ur response.....i choose....ask transalator to ask mother abt coin practice ...the qs abt folk medicine. 20/ h/o ASD(IN HISTORY fixe split s2...) 21/h/o photosenstivity.......side effect...doxycyclin. 22/ thr is a history....pat was on methotrexate...prednisone...they wr asking abt wat shld we need to ad more...... i choose cox 2 inhibitor coz... prdnisone increaase expression of cox 2 so patient can have inflammatory side effects thats the reason i choose cox 2 i dont know its right ans or not. 23/h/o nulligravida.......risk of...ans endometrial CA 24/in clinical study ...if u find a pt with MI wat u will find in ECG....i choose st segment depreesion,,,i look for st segment elevation but i din find so i end with this ans. 25/h/o married couple they were tring to have baby since 2 months,ther sexual practise normal,,they both have orgasm normal, but they fight abt the baby prob so coz of it women is depress but at work she is normal.......a sexaul arousal prob,b dysthymia, c some sexual prob i don remmebr.but i choose dysthymia .i don no its ryt or rong. 26cohort 26/methoterexate 27/h/o plantar fascitis ....pt was casher ...tenderness sole of foot from heel to front.. 28/h/o long trem pain prob....had gone thru lots of dignostic measures

http://www.usmleforum.com/files/forum/2007/1/193828.php[11/7/2011 2:10:26 PM]

Remembered questions for STEP 1 - USMLE Forum but stil have same prob.....came to doc saying i thing u r fooling me...wat ur response......a. u can switch other doc,,b i will refer to pscycgh...c, i am doing alll appropriate measures in order... this one i choose 29/2 qs abt acid base 30/ patient was on high alltitude.....now at sea level wat will be PO2,PCO2,and PH. 31/INSULIN qs.....h/o post prandial it was in grapl 32/secretin in graph 33/pt have prob in swallowing ....had surgerythoracic....mass in mediastinum ,,,removed...wat will intact after surgery.a.lower esophageal sphincter response..some more choices i don remmeber. 34/a pt hit by a truck...rear.......?a.direct injury.d hyperextension injury,,c hypoextension ..i choose direct .. 35/8 yr bouy had accident ....sitting front....got multiple fracture,,,no hope of survival....wat ur respone to parents.a. it must be moral 4 u guys. thr some more but stupid ans,,,,i choose the one which has assurance i dont remmber the choice . 36/77 yr old lady was on dialysis since 10 yrs she was tired of it want no more... but she was asking that am i have painful death wat ur response...............u will have some fluid retention but we r here we l do our best possible to alleviate ur problem. 37/chlamadia 38/oxidase pos....psedomonas 39/latral pontine syndrome 40/clozapine toxi 41/occulomotor 2 qs 42/hemisection cord 2 qs 43/UMN lesion 44/upper quadrantopnia....middle cerebral artery 45/picI...arrow was on choroidal artery 46/cerebral artery 47/antihistamine 2 qs 48/teenager boy who was normal,play with friends often,but now mother complaing that he sleeps althe time.not playing with pals,no dru abuse wat RX a chlodiazepoxide b flostine.c resperidone 49/ ANS MEDUIM CHAIN (MCAD)///NO KETONE BODIES 50/ urine methylmalonic acid...... 51/tb patient orange yellow urine....sweat...eyes...ans rifampin 52/qs 19 yr old house....urine inc ALA 51/cholyStyramin MOA 52/LDL receptor easy history 53/46 yr old male crying that i will die early like my father?wat ur response/....i choose tell me abt ur father ....how he died. 54/dabetic pt not taking care....follow up vist wat ur response......ii choose tell me wat u know abt dibetes 55/another diabetic patient ,,taking good care of diabetes,,exersice daily....wat ur advise in end a.pres vit E.,b fish oil.,c check ur feet every day. 56/kidney stone pat ....wat advice u will give 57/obese diabetic pt wat treatment....a,insulin....,b,glyburide,....cmetformin 58/obese pt ...gall stone...u find in ultrasound...wat wiill be next mangement.a.uricodxycolic acid.b,open stomech remove stone.,c endoscopic removal 59/chrons Rx..salfasalzine 60/chemo pt nausea vomiting...rx ondansatren' 61/moa thiazoide 62./in er pt have mi wat tx... nitrogylycerine. b beta blocker. c dgixin 63/broad ligament of uterus 64/alcholic......acut pancreatitis 65/marfan pt....cystic medil necrosis 66/substance abuse........all nihgt dancing....ans majuana 67/xray fracture arrow on navicular bone(which i did rong i m upset i knew tat but still picked wrong) 68/xray/....sholder......arrow on biceps brachi 69/wiskot aldrich 70/LB4 71/side effect of EFAVERINZE(rash history) 72/bcl2.....folicular lymphoma 73/blast crisis........cml 74/obese pt sleep apnea.......dignosiss?how 75/ca mechanism ON SR

http://www.usmleforum.com/files/forum/2007/1/193828.php[11/7/2011 2:10:26 PM]

Remembered questions for STEP 1 - USMLE Forum 76/colon CA ....MISMATCH REPAIR 77/membranous glomerulonephritis 78/pt ANA positive... ig G,C3 DEPOSIT....good pature..acute gn 79/women ...4 wt loss.....which drug.....amphatemine 80/ADH SOME GRAPH ON IT 81/sarcodosis...noncaseating granuloma 82/primaquin moa 83/MIF (MULLERIN INHIBITORY FACTOR)MUTATION....... 84/qs abt renin ....graph 85/no spleen....risk.... strep pnemon.. 86/77 yr pt which vaccine......i choose hinfulenza vaccine& pnemo vaccine 87/seminoma 88/staph /strep....ow diff ....ans catalase 89/p value was less then 1.....a.rejection of true null hypothesis.,b tejection of false null hypothesis 90/tuberssclerosis 91/compititive inhibitor graph 92/h'/o paranoid 93/plasmid bacteria.........antibiotic resistance 94/dorsal plantar arch 95/cor pulmo.......S3 96/benzo......MOA 97/polyuria ...which drug....lithium 98/pt feeding milk......dry vagina.....y?a.prolactin,more choice i don rember 99/rx vit k in warfarin tox 100/factor 2 def.......whr prob.......liver,heart,lung,breast 101/GnRH.....RX IN.... 102/estrogen......ingraph 103/sildenafil side effect 104/allupurinol moa 105/cycloserine moa 105/copd pt .......in graph 106/asking to calculate stroke volume 107/fibrocystic change......with cycle 108/3 qs on lober pnmo 109//acute pancreatitis....coz....gall stone 110/ketone bodies...............beta hydroxybutyrate 111/slide of bone formation 112/ribosomal rna synthesis 113/osteogenesis imperfecta........procollagen prob 114/qs abt st jhon wart medication 115/asprin toxi....rx 116interferon moa 117/calculate mean...in graph describing kids having uti. 118/thr was control grp versus disease in grph ...which one have 5 yr more survival then control. 119/elongation step in tranlation 120/moa ondinavir 121/HTLV1 122/injury at L5 123/HARDY WEIN BRG.....1/1000...SOME THIN LIKE TAT 124/WERD PIC ASKING ABT WHR BACTERIA IS INFECTING. 125/CALCULATE TOATL BODY WATER 126/XRAY/KNIFE...NEAR RT VENTRICAL OF HEART 127/HOMEO BOX MUTATION...INSTEAD HEAD LEG ...SOME WEIRD THING 128/ALLOTYPE DIAGRAM 129/INH TOX • infant in carseat------- face to rear *odd ratio *wiskott -aldrish *MOA of Antidote in phophate intoxication Report Abuse

http://www.usmleforum.com/files/forum/2007/1/193828.php[11/7/2011 2:10:26 PM]

Remembered questions for STEP 1 - USMLE Forum

* Re:Remembered questions for STEP 1

#1119522

mysam - 01/10/08 02:47

bump!! Report Abuse

Page 1 of 1 []

Logon to post a new Message/Reply

Search

Web

Step 1

Step 2 CK Login

USMLEforum.com

Step 2 CS

Matching & Residency

USMLE Links

http://www.usmleforum.com/files/forum/2007/1/193828.php[11/7/2011 2:10:26 PM]

Step 3

Miscellaneous Home

View more...

Comments

Copyright ©2017 KUPDF Inc.
SUPPORT KUPDF